Categories
Economic History Exam Questions Fields Harvard Statistics

Harvard. Division Exams for A.B., General and Economics, 1920

The Harvard Economics department was once one of three in its Division in the Faculty of Arts and Sciences. The Departments of History and Government shared a general division exam with the Department of Economics and also contributed their own specific exams for departmental fields. This post provides the questions for the common, i.e. general, divisional exam and all the specific exams at the end of the academic year 1919-20 for fields covered by the economics department.

_______________________

Previously Posted Division A.B. Exams from Harvard

Division Exams 1916

Division Exams, January 1917

Division Exams, April 1918

Division Exams May 1919

Division Exams 1931

Special Exam for Money and Government Finance, 1939

Special Exam Economic History Since 1750, 1939

Special Exam for Economic Theory, 1939

Special Exam for Labor and Social Reform, 1939

_________________________

DIVISION OF HISTORY, GOVERNMENT AND ECONOMICS

DIVISION EXAMINATIONS FOR THE DEGREEE OF A.B.
1919-20

DIVISION GENERAL EXAMINATION
[April 29, 1920.]

PART I

The treatment of one of the following questions will be regarded as equivalent to one-third the examination and should therefore occupy one hour. Write on one question only.

  1. Compare pamphleteering and propaganda as methods of exerting political influence.
  2. What effect has the establishment of standing armies and navies had upon (a) political and (b) economic organization of the state?
  3. Show how, and why, the following were adapted to certain stages of society: (a) feudalism; (b) gilds; (c) nationality; (d) industrialism.
  4. Trace the course and explain the significance of the development of maritime law.
  5. Contrast the Greek and Italian city states.
  6. What are the wastes of the present industrial system and how, if at all, are they to be eliminated?
  7. Comment on the following: “History embraces ideas as well as events, and derives its best virtues from regions beyond the sphere of state.”
  8. Discuss the problems involved in the economic rehabilitation of Central Europe.
  9. What are the rights of minorities and how are they best secured?
  10. Compare the foreign policies of France, Germany, and the United States during the nineteenth century.

PART II

The treatment of three of the following questions in Part II is required and will be regarded as equivalent to one-third of the examination, and should therefore occupy one hour. The three questions are to be taken from the Departments in which the student IS NOT CONCENTRATING; two questions from one of these Departments and one question from the other.

A. HISTORY

  1. Why did Voltaire characterize the Holy Roman Empire as “neither holy, nor Roman nor an Empire”?
  2. What do you regard as the six most important naval battles in the history of the world! When and where were they fought, and who were the victors and the vanquished in each?
  3. Give a brief account of the relations of the United States and Canada.
  4. What have been the principal issues involved in the struggle over Home Rule?

B. GOVERNMENT

  1. What was the political condition of European states at the time of the Crusades!
  2. In what sense are constitutions of states “made”?
  3. If the principle of reparation of governmental powers is correct, why has the English cabinet system been approved?
  4. Explain the reasons for immigration to the United States from 1870 to 1895.

C. ECONOMICS

  1. What has been the contribution of the corporation to English and American political and economic institutions?
  2. Trace the evolution of collective bargaining in industry.
  3. What is “profiteering”? Explain its relation to the present high cost of living.
  4. Describe the development, and indicate the importance, of national budgets.

PART III

The treatment of three of the following questions in Part III is required and will be regarded as equivalent to one-third of the examination, and should therefore occupy one hour. The three questions are to be taken from the Department in which the student IS CONCENTRATING.

A. HISTORY

  1. Describe the changes in the attitude towards the Christians of the Roman Emperors down to Constantine.
  2. Discuss the development of national assemblies during the Middle Ages.
  3. What did the Tudors do for England?
  4. What is now the territory within the jurisdiction of the United States has been derived, directly or indirectly, from seven European nations. What are the seven, and what territory was derived from each?
  5. Enumerate, with dates, the principal changes in the form of government of France since 1789. How do you account for their frequency?

B. GOVERNMENT

  1. Discuss the development of the relations of President and Cabinet in the United
  2. Discuss and illustrate the following: “If tolerance can be allowed in a state, so much the better; that proves that the state is strong.”
  3. What should be the disposition of Constantinople?
  4. Give a brief sketch, explaining cause and naming period, of three of the following: (a) Dorr Rebellion; (b) Whiskey Insurrection; (c) Shay’s Rebellion; (d) Seminole War; (e) Ku-Klux Klan.
  5. How has the change in distribution of population affected governmental organization and administration?

C. ECONOMICS

  1. Discuss the probable future of the market for loanable funds in the United States and Europe.
  2. State the purposes and proper limits of progressive taxation.
  3. Describe the efforts of the Federal Government to enforce fair competition.
  4. What considerations are involved in the maintenance of public agencies for the distribution and employment of labor? What light is thrown on the subject by American and European experience?
  5. Sketch the history and present prospects of the American merchant marine.

_________________________

DIVISION OF HISTORY, GOVERNMENT, AND ECONOMICS

DIVISION SPECIAL EXAMINATION
ECONOMIC THEORY
[May 3, 1920.]

Answer six questions

A

Take from this group at least two and not more than four

  1. “It is the business of economic theory to explain, not to justify or condemn.” Comment.
  2. Discuss the rôle of mathematical analysis in the development of economic theory.
  3. “The determining cause of the general rate of money incomes and wages in a country is to be found in the exporting industries.” Explain.
  4. “The income from concrete instruments of production may be regarded as ‘rent’ or as ‘interest’ according to the point of view.” Explain and discuss.
  5. Of what concretely do invested, of what do uninvested, savings consist? Can savings accumulate to an indefinitely large amount? Can saving be carried to excess?
  6. “The standard of living affects wages, not directly, but through its influence on numbers. … A limitation of numbers is not a cause of high wages, but it is a condition of the maintenance of high wages.” Explain and criticize.
  7. Discuss the theories of business profits.

B

Take from this group at least one and not more than two

  1. Outline the history of mercantilism.
  2. Give an account of an important political episode in which economic theory has had a decisive influence.
  3. Trace the course of the rate of interest in modern times. What do you expect to be the course of the rate during the next fifteen years? Why?
  4. Characterize the more important developments in the history of socialism.

C

Take from this group at least one and not more than two

  1. What is the relation of (a) investment banking, (b) commercial banking, to capitalistic production?
  2. What theoretical problems are involved in government regulation of security issues?
  3. Does profit-sharing promise a solution of the problems of distribution? Why or why not?
  4. Discuss the following statement: “If you are not advertising, then advertise, because it saves money for you and it reduces the price to the consumer.”

_________________________

DIVISION OF HISTORY, GOVERNMENT, AND ECONOMICS

DIVISION SPECIAL EXAMINATION
ECONOMIC HISTORY
[May 3, 1920]

Answer six questions

A

Take from this group at least one and not more than two

  1. To what extent, if at all, and in what particulars, has the policy of high protection been of advantage to the American laborer?
  2. How do price revolutions, such as that in progress since 1897, tend to affect the distribution of wealth?
  3. Briefly explain the most satisfactory statistical methods for separating the different types of variation in time series.
  4. What is a logarithmic curve? What are its merits and defects in the graphic presentation of historical series?
  5. Trace the development of uniform accounting for railroads in this country. Indicate any connections between uniform accounting and government regulation of the railroads.

B

Take from this group at least two and not more than four

  1. Discuss the economic results of the crusades.
  2. Give a brief historical account of mercantilism.
  3. Outline the history of the public debt of one of the following countries: (a) Great Britain; (b) France; (c) United States.
  4. Trace the agrarian movement on the continent of Europe.
  5. Discuss the positions of the various English political parties and social classes on the question of Corn Law Repeal.
  6. Write a brief history of one of the following industries in the United States:

(a) Meat-packing;
(b) Tin-plate manufacture;
(c) Boot and shoe manufacture;
(d) Ship-building.

  1. When and by what steps was silver demonetized in the United States?
  2. Outline the development of the English textile industry.
  3. Give a brief account of the “trust movement” in the United States since 1898.
  4. Sketch the history of the export trade of the United States.

C

Take from this group at least one and not more than two

  1. Analyze the effects of England’s early commercial policy.
  2. What specific defects in the National Banking System was the Federal Reserve Act, 1913 intended to remedy?
  3. Trace and explain the history of the American merchant marine since 1840. What is its probable future and why?
  4. What industrial conditions are most conducive to the rapid growth of labor organizations? Why?

_________________________

DIVISION OF HISTORY, GOVERNMENT, AND ECONOMICS

DIVISION SPECIAL EXAMINATION
MONEY AND BANKING
[May 3, 1920]

Answer six questions.

A

Take from this group at least one and not more than two

  1. Discuss the distinction between currency expansion and currency inflation.
  2. What statistics of money and banking best serve as indices of financial, speculative, and general business conditions?
  3. Outline a system of accounts for a small commercial bank.
  4. What are the best sources of statistical data upon the following subjects:

(a) Bank clearings in the United States;
(b) Resources and liabilities of banks of New York City;
(c) Bank rates in the London and Paris money markets;
(d) The monetary stock of the United States;
(e) Changes in the value of gold in England?

B

Take from this group at least one and not more than two

  1. Outline the currency history of one of the following:

(a) Canada;
(b) Germany;
(c) British India;
(d) the Philippines;
(e) the American colonies;
(f) Russia.

  1. State and explain Gresham’s Law. Give four historical examples of the working of the law.
  2. Sketch the history of the relations between the United States Treasury and the banking institutions of the country.
  3. Compare American, British, and German banking methods and policy during the World War.
  4. Describe in detail one of the following financial panics: 1837; 1873; 1893; 1907.

C

Take from this group at least two and not more than four

  1. What have been the causes of the rehabilitation of silver?
  2. What are the arguments for and against an embargo upon gold exports from the United States at this time?
  3. Describe the business of an American bond house.
  4. Discuss critically the following statement made early in 1916:
    “The recent enactment of the Federal Reserve Act only made our sudden riches more embarrassing, for that Act had so changed our system of banking that every $18 of gold in the banks created $82 worth of loanable credit, whereas formerly, of every $100, $25 had to sit in the vaults while only $75 went out to work in the form of loans. In other words (as a result of the War and our banking reform), we not only had enormously more gold, but every dollar of it went a good deal further than ever before in financing new enterprises. This is the situation today.”
  5. Give a critical analysis of the policies of the Federal Reserve Board.
  6. Compare banking in France and England since the signing of the armistice.
  7. Why has London been the financial center of the world? What are the prospects that New York will in time displace London?

_________________________

DIVISION OF HISTORY, GOVERNMENT, AND ECONOMICS

DIVISION SPECIAL EXAMINATION
CORPORATE ORGANIZATION, INCLUDING TRANSPORTATION
[May 3, 1920]

Answer six questions

A

Take from this group at least one and not more than two

  1. In discussing the problems of capitalistic monopoly, it has been stated that “the matter at issue is a question, less of relative ‘economy’ of monopoly and competition than of the kind of economic organization best calculated to give us the kind of society we want.” Explain and discuss.
  2. What are the methods of measuring depreciation? What different policies with respect to depreciation have been advocated in the regulation of public utility rates?
  3. Discuss comparatively the public regulation of railway accounts in England, France, and the United States.
  4. To what extent do the reports of the Bureau of the Census furnish data upon corporate enterprise in the United States?

B

Take from this group at least one and not more than two

  1. Sketch the history of the Sherman Anti-Trust Law and its enforcement.
  2. Give a brief account of the functions and work of the United States Bureau of Corporations.
  3. Trace the evolution of the equipment of the modern railway.
  4. Outline the history of railroads in Germany.

C

Take from this group at least two and not more than four

  1. What are the purposes and customary scope of “blue sky” laws? What is the case for and against such legislation?
  2. What connections exist between banks and industrial combinations in the United States? Contrast the situation here with that in France.
  3. Compare American and German public policy toward industrial combinations.
  4. Give a critical analysis of the present railway rate structure in the United States.
  5. Discuss the Plumb Plan for the ownership and operation of the railways of the United States.
  6. Discuss the effects of the great inter-oceanic canals upon inland and ocean transportation.
  7. What are the problems of excess profits taxation?

_________________________

DIVISION OF HISTORY, GOVERNMENT, AND ECONOMICS

DIVISION SPECIAL EXAMINATION
PUBLIC FINANCE
[May 3, 1920]

Answer six questions.

A

Take from this group at least one and not more than two

  1. To what extent, if at all, and in what manner, are taxes a contributing cause of the present “high cost of living”?
  2. Discuss the proposal to tax individuals in proportion to their expenditure rather than their income, thus exempting savings.
  3. Describe the statistical features of the Census Bureau’s annual reports on “Financial Statistics of Cities.”
  4. What course has been taken by the reform of municipal accounting in the United States?

B

Take from this group at least one and not more than two

  1. Outline the development of the science of public finance.
  2. Give a critical account of the Independent Treasury of the United States.
  3. Trace the history of budget plans in American state and municipal government.
  4. Compare the financing of the American and French Revolutions.
  5. Give a brief historical account of direct taxation in Germany.
  6. Develop and defend a classification of public revenues.

C

Take from this group at least two and not more than four

  1. Compare government monopolies and internal revenue taxes as means of raising national funds.
  2. Analyze the financial results of the operations of the United States Post Office.
  3. Upon what bases should public utilities be valued and paid for when taken over by municipal authorities?
  4. “Taxation, while necessarily involving political and social considerations, is essentially a problem in national economies.” Do you agree? State your reasons.
  5. The practice of exempting government bonds from taxation is a pernicious American custom.” Comment.
  6. Discuss the effects of national prohibition upon public finance.
  7. Give a critical analysis of excess profits taxation.

_________________________

DIVISION OF HISTORY, GOVERNMENT, AND ECONOMICS

DIVISION SPECIAL EXAMINATION
LABOR PROBLEMS
[May 3, 1920]

Answer six questions

A

Take from this group at least one and not more than two

  1. Discuss the causes of the prevailing industrial unrest.
  2. To what extent and for what purposes should the state limit the hours of labor?
  3. Describe the technique of analyzing workingmen’s budgets.
  4. What statistical problems are involved in measuring labor turnover? What methods of measurement are most satisfactory?

B

Take from this group at least one and not more than two

  1. Compare trade unions and trade gilds, and the industrial conditions under each.
  2. Give a brief historical account of the employment of children in industry.
  3. Outline the development of the Railway Brotherhoods.
  4. Trace the history of the German Social Democratic Party.

C

Take from this group at least two and not more than four

  1. Analyze labor conditions in one of the following industries: (a) cotton manufacture; (b) coal mining; (c) steel manufacture.
  2. Discuss the main points of economic policy in the “reconstruction program” of the British Labor Party.
  3. What is the extent and importance of industrial unemployment?
  4. Discuss the present status of women in industry.
  5. Characterize the organization and results of the Washington industrial conferences of 1919-20.
  6. Discuss the aims, scope, and methods of employee representation in business management.
  7. What public policy should be adopted in regard to labor organizations among government employees?

_________________________

DIVISION OF HISTORY, GOVERNMENT, AND ECONOMICS

DIVISION SPECIAL EXAMINATION
ECONOMICS OF AGRICULTURE
[May 3, 1920]

Answer six questions

A

Take from this group at least one and not more than two

  1. To what extent are wages of management an element of cost in American agriculture?
  2. What are the interrelations of cold storage and prices of farm products?
  3. What statistical records are desirable for efficient operation of a dairy farm?
  4. To what extent and in what particulars is depreciation involved in farm accounting?

B

Take from this group at least one and not more than two

  1. Describe the agrarian revolution in England.
  2. Sketch the movement of the wheat belt in the United States since colonial times.
  3. Give a brief historical account of farm tenancy in the United States.
  4. Trace connections between the tariff policy of the United States and wool growing in this country.
  5. Outline the development of the work of the United States Department of Agriculture.

C

Take from this group at least two and not more than four

  1. Discuss the relations between climate and the productivity of land.
  2. Indicate the origins of the more important breeds of live stock. What contributions, if any, has this country made to the improvement of the breeds?
  3. Describe the effects of the World War upon the wool market.
  4. What are the relations between the wages of agricultural and factory labor?
  5. Compare the use and importance of artificial fertilizers in American and European agriculture.
  6. Give a brief critical analysis of the Federal Farm Loan Act.
  7. What are the causes of the increasing urbanization of population in the United States?

_________________________

DIVISION OF HISTORY, GOVERNMENT, AND ECONOMICS

DIVISION SPECIAL EXAMINATION
STATISTICS
[May 3, 1920]

Answer six questions

A

Take from this group at least two and not more than four

  1. Draft a set of rules for the construction of statistical tables.
  2. Explain “the necessity of the logical agreement of magnitudes from which an average is to be computed” and compare this with the requirement of “the greatest possible homogeneity of series.” Which of these requirements seems to you more difficult of fulfilment? Why?
  3. Describe the short-cut method of calculating the arithmetic mean from a frequency table. What assumptions underlie this method?
  4. Explain briefly: (a) discrete series; (b) mode; (c) Lorenz curve; (d) average of position; (e) Galton graph.
  5. Explain the different methods of eliminating secular trend in historical series.
  6. Describe the construction and characteristics of a logarithmic curve. What are the merits and defects of such a curve?
  7. What are the comparative advantages and disadvantages of chain indices and fixed-base indices in the measurement of changes in the price level?

B

Take from this group at least one and not more than two

  1. Give a brief account of the evolution of Statistics.
  2. Outline the history of the Bureau of the Census.
  3. Trace the development of price statistics in England.
  4. What has been the history of wage statistics in the United States?

C

Take from this group at least one and not more than two

  1. Discuss the different methods employed in estimating population.
  2. What are the principal difficulties in the collection of mortality statistics?
  3. Discuss critically current statistics of foreign trade in this country and abroad.
  4. What units have been employed in the statistics of railways? Analyze and appraise the different units.
  5. What are the best sources of statistical data upon the following subjects:

(a) Bank clearings in the United States;
(b) Resources and liabilities of banks of New York City;
(c) Bank rates in the London and Paris money markets;
(d) The monetary stock of the United States;
(e) Changes in the value of gold in England?

 

Source: Harvard University Archives. Harvard University Divisional and General Examinations, 1915-1975. Box 6, Bound volume Divisional Examinations 1916-1927 (From the Private Library of Arthur H. Cole).

Image Source: Widener Library from Harvard Class Album 1920.

Categories
Harvard Statistics Suggested Reading

Harvard. Short Bibliography on Social Statistics for “Serious-minded Students”, Ripley, 1910

 

In 1910 Harvard published a total of 43 of short bibliographies in the collection “Social Ethics and Allied Subjects”, about half of which were dedicated to particular topics in economics and economic sociology. The project was coordinated by Plummer Professor of Christian Morals, Francis G. Peabody.

Part I, Chapter 7 “Social Statistics” was the first of economics professor William Z. Ripley’s three contributions to the collection. Most items have been conveniently linked to internet archival copies for this post.

Previously posted bibliographies from Peabody’s “Social Ethics and Allied Subjects”:

Economic Theory by Professor Frank Taussig

Taxation by Professor Charles J. Bullock

Trade Unionism by Professor William Z. Ripley

Social Insurance by Dr. Robert Franz Foerster

Economics of Socialism by Professor Thomas Nixon Carver

Strikes and Boycotts by Professor William Z. Ripley

Socialism, Family and Religion by Dr. Ray Madding McConnell

______________________

7. SOCIAL STATISTICS

William Z. Ripley

The principal source for the United States is the Federal Census,” taken decennially since 1790. The later ones are summarized in special volumes, that for 1900 being known as the Supplementary Analysis.” A still briefer Abstract” is also published. The Supplementary Analysis” is, however, more serviceable, inasmuch as it offers in the text a critical examination of the figures, setting forth the limitations upon their use. A Statistical Atlas” also accompanies each census since 1870, offering the results in graphic form. Special reports” on a variety of topics are also issued. Among these since 1900 may be mentioned, A Century of Population Growth,” Washington, 1909; Marriage and Divorce,” 1867–1906, two parts [Part One Summary, Laws, Foreign Statistics; Part Two, General Tables], Washington,1909, and Statistics of Cities.” “Mortality Statistics” for all states having registration laws are also compiled and published annually. This is the standard reference for all states and minor subdivisions.

Several of the Commonwealths, notably Massachusetts, take censuses on the intervening quinquennial years, some of them containing more detailed information than is afforded by the federal census. The United States Statistical Abstract,” published annually [1878-2018], is often serviceable.

For foreign countries, the Statistical Abstract for Great Britain” and also “The Statistical Abstract for the Principal and other Foreign Countries,” will serve as convenient and accurate handbooks. Some of the leading newspapers, notably the New York Tribune [1856-1914] and the World, [1868-1925], publish Almanacs,” containing a wide range of material gathered from official publications. The Statesman’s Year Book” [1876-1923] is a valuable source. The most elaborate scientific collection, issued annually and covering a very wide range of social data, is, G. Sundbärg: Aperçus statistiques internationaux.” Stockholm: Imprimerie Royale. Since 1906 this has been published in French.

Among scientific serials which are invaluable to students of social statistics, not only for their separate articles, but for their reviews of current literature, should be included: Quarterly Publications of the American Statistical Association [1888-1921] and Journal of the Royal Statistical Society.

The following standard treatises are of fundamental importance, containing not only positive statistical data, but critical examination as to their value:

Mayo-Smith, Richmond. Statistics and sociology. New York: The Macmillan Company, 1895, pp. xvi, 399.

Mayo-Smith, Richmond. Statistics and economics. New York: The Macmillan Company, 1899, pp. 467.

These volumes of material up to the date of their publication are most valuable. They are, however, less ample in American material than for foreign countries.

Bailey, William B. Modern social conditions. New York: The Century Company, 1906, pp. 377.

This treatise is confined principally to the field of vital statistics and problems of population.

Bowley, Arthur Lyon. Elements of statistics. London: P. S. King & Son, 1901, pp. 328.

The most scientific treatise extant as to statistical method, but less serviceable as a collection of positive data. [First edition (1901); Second edition (1902); Third edition (1907); Fourth edition (1920); Fifth edition (1926); Sixth edition (1937)]

Wright, Carroll D. Outline of practical sociology. New York: Longmans, Green & Co., 1899, pp. XXV, 431.

This obviously does not include data from the census of 1900.

Newsholme, A. Elements of vital statistics. Third edition. London: Swan, Sonnenschein & Co., 1899, pp. xxiv, 326.

The standard authority for this special field.

Walker, Francis A. Discussions in economics and statistics. 2 vols. New York: Henry Holt & Co., 1899, pp. iv, 454, 481. [Volume 1, Finance and Taxation, Money and Bimetallism, Economic Theory; Volume 2, Statistics, National Growth, Social Economics]

Weber, Adna F. The growth of cities in the nineteenth century. New York: Columbia University Studies, 1899, pp. xiv, 497.

This work is a convenient source book for all statistics of cities.

Schnapper-Arndt, Gottlieb. Sozial-Statistik. Leipzig: W. Klinkhardt, 1908, xxii, 642 S.

This book offers a readable account of statistical data, mainly for European countries.

Mayr, Georg von. Statistik und Gesellschaftslehre. 3 Bde. Freiburg i. B.: J. C. B. Mohr, 1895-1909, 202, 486, 260 S.  [Volume 1, Theoretische Statistik; Volume 2, Bevölkerungsstatistik; Volume 3, Sozialstatistik]

The most elaborate treatise, serviceable mainly to specialists. It contains a wonderfully complete bibliography in all languages, arranged under separate headings, which may be of use in looking up special topics.

Source: A Guide to Reading in Social Ethics and Allied Subjects. Lists of books and articles selected and described for the use of general readers by teachers in Harvard University. Cambridge, Massachusetts (1910), pp. 32-34.

Image Source: Harvard University Archives. William Zebina Ripley [photographic portrait, ca. 1910], J. E. Purdy & Co., J. E. P. & C. (1910).

Categories
Harvard Statistics Suggested Reading Syllabus

Harvard. Graduate Econometrics. Chipman 1953

 

John S. Chipman was born in Montreal, Canada, in 1926. He received his Ph.D. from the Johns Hopkins University in 1951, and taught at the University of Minnesota from 1955 to his retirement as Regents’ Professor in 2007. Before going to Minnesota, Chipman was assistant professor of economics at Harvard from 1951-55.

John Chipman’s graduate course in econometrics (Note: apparently the first time “econometrics” had ever been listed as the course name at Harvard) followed his graduate course on “General Interdependence Systems”  that was taught in the spring term 1952 (a.k.a. “Mathematical Economics” according to the enrollment data for that year). 

____________

Course Enrollment

[Economics] 215 Econometrics. Assistant Professor Chipman. Half course.

(Fall) Total 6: 5 Graduates, 1 Other.

Source: Harvard University. Report of the President of Harvard College 1953-1954, p. 100.

____________

Course Syllabus
Economics 215

HARVARD UNIVERSITY
Department of Economics
Fall Term, 1953-54

Text: Klein, A Textbook of Econometrics; Supplementary text: Hood and Koopmans, Studies in Econometric Method.

(N.B. Numbers in brackets refer to References following the Outline).

OUTLINE

1. SPECIFICATION

The construction of models, choice of variables; collinearity; concepts of structure and reduced form; aims and purposes of econometrics.

Jacob Marschak, “Economic Measurements for Policy and Prediction,” [3, Ch. I].

Haavelmo [2].

Stone [8].

Tinbergen [10, 9].

Marschak, “Statistical Inference in Economics: An Introduction,“ [6, Ch. I].

Tintner [12, Ch. I].

Arthur F. Burns and Jacob Marschak, “Mitchell on What Happens During Business Cycles,” [7, pp. 3-33].

Tinbergen and Koopmans, “Reformulation of Current Business Cycle Theories as Refutable Hypotheses,” [7, pp. 131-145].

Frisch [1].

Koopmans [5].

Tinbergen [11].

Koopmans, “Measurement Without Theory,” Review of Economic Statistics, Vol. 29, 1947.

Koopmans and Vining, “Methodological Issues in Quantitative Economics”, Review of Economics and Statistics, Vol. 31, 1949.

G.H. Orcutt and Others, “Toward Partial Redirection of Econometrics,” Review of Economics and Statistics, August 1952.

G.H. Orcutt, “Actions, Consequences, and Causal Relations,” Review of Economics and Statistics, November 1952.

Klein [4, Ch. I].

 

2. IDENTIFICATION

The problem of determining parameters of a system of structural equations given the parameters of the reduced form equations.

Koopmans, “Identification Problems in Economic Model Construction” [3, Ch. II].

Klein, [4, Ch. III, Sec. 3].

Simon, “Causal Ordering and Identifiability” Klein, [3, Ch. III].

Koopmans and Reiersøl, “The Identification of Structural Characteristics,” [C.C.P. no. 39]

Koopmans, Rubin, and Leipnik, “Measuring the Equation Systems of Dynamic Economics,” [6, Ch. II, Sec. 3].

Hurwicz, “Generalization of the Concept of Identification” [6, Ch. IV].

 

3. ESTIMATION

Point estimation of reduced-form parameters and structural parameters; statistical independence of observations in time series and cross-section data; assumptions of normality in joint distribution of disturbances; principles of estimation; maximum likelihood, limited information, least squares.

Klein [4, Ch. III].

Koopmans and Hood, “The Estimation of Simultaneous Linear Economic Relationships,” [3, Ch. VI].

Koopmans, Rubin, and Leipnik, “Measuring the Equation Systems of Dynamic Economics,” [6, Ch. II, Sec. 3].

Haavelmo, “Methods of Measuring the Marginal Propensity to Consume,” [3, Ch. IV].

Girshick and Haavelmo, “Statistical Analysis of the Demand for Food,” [3, Ch. V].

Anderson and Rubin [C.C.P., No. 36].

D. Cochrane and G.H. Orcutt, “Application of Least Squares Regression to Relationships Containing Auto-Correlated Error Terms,” Journal of the American Statistical Association, March 1949.

G.H. Orcutt and D. Cochrane, “A Sampling Study of the Merits of Autoregressive and Reduced Form Transformations in Regression Analysis,” Journal of the American Statistical Association, September 1949.

Anderson and Anderson [C.C.P. No. 42].

Klein [4, Ch. V].

F.N. David and J. Neyman, “Extension of the Markoff Theorem on Least Squares,” Statistical Research Memoirs, Volume II, December 1938.

T.W. Anderson, “Estimation of the Parameters of a Single Equation by the Limited-Information Maximum-Likelihood Method” [6, Ch. IX].

Chernoff and Rubin, Asymptotic Properties of Limited-Information Estimates Under Generalized Conditions,” [3, Ch. VII].

Jean Bronfenbrenner, “Sources and Size of Least-Squares Bias in a Two-Equation Model,” [3, Ch. IX].

 

4. VERIFICATION

Interval estimation and testing of hypotheses.

Klein [4, Ch. 3, Sec. 6].

Haavelmo [2, Ch. IV].

Carl Christ, “A Test of an Econometric Model for the United States, 1921-1947,” [7, pp. 35-129].

Anderson [C.C.P. No. 50].

Tinbergen [10].

 

5. PREDICTION

Conditional and unconditional prediction; relationship between prediction and policy.

Klein [4, Ch. VI].

Haavelmo [2, Ch. VI].

Hurwicz, “Prediction and Least Squares,” [6, Ch. VI].

Tinbergen [11].

REFERENCES

1. PRINCIPLES

Books and Monographs:

[1] Frisch, Ragnar: Statistical Confluence Analysis by Means of Complete Regression Systems, Oslo, Universitets Økonomiske Institut, 1934.

[2] Haavelmo, Trygve: “The Probability Approach in Econometrics,” Econometrica, Vol. 12 (Supplement), July 1944.

*[3] Hood, Wm.C., and Koopmans, T.C., (ed.): Studies in Econometric Method, Cowles Commission Monograph No. 14, New York, Wiley, 1953.

*[4] Klein, L.R.: A Textbook of Econometrics, Evanston, Illinois, Row, Peterson and Co., 1953.

[5] Koopmans, T.C.: Linear Regression Analysis of Economic Time Series, Haarlem, De Erven F. Bohn N.V., 1937.

[6] Koopmans, T.C. (ed.): Statistical Inference in Dynamic Economic Models, Cowles Commission Monograph No. 10, New York, Wiley, 1950.

[7] National Bureau of Economic Research: Conference on Business Cycles, New York, 1951.

[8] Stone, Richard: The Role of Measurement in Economics, Cambridge University Press, 1951.

[9] Tinbergen, Jan: Econometrics, Philadelphia, Blakiston, 1951.

[10] Tinbergen, Jan: Statistical Testing of Business Cycle Theories, Geneva, League of Nations, 1939.

[11] Tinbergen, Jan: On the Theory of Economic Policy, Amsterdam, North-Holland Publishing Co., 1952.

[12] Tintner, Gerhard: Econometrics, New York, Wiley, 1952.

Cowles Commission Papers (New Series):

No. 36. T.W. Anderson and Herman Rubin, Two Papers on the Estimation of the Parameters of a Single Equation in a Complete System of Stochastic Equations, Annals of Mathematical Statistics, Volume 20, 1949 and Volume 21, 1950.

No. 39. Olav Reiersøl and Tjalling C. Koopmans, Three Papers on Identification Problems, Psychometrica, Volume 15, 1950; Annals of Mathematical Statistics, Vol. 21, 1950; and Econometrica, Vol. 18, 1950.

No. 42. R.L. Anderson and T.W. Anderson, “Distribution of the Circular Serial Correlation Coefficient for Residuals from a Fitted Fourier Series,” Annals of Mathematical Statistics, Volume 21, 1950.

No. 50. T.W. Anderson, “Estimating Linear Restrictions on Regression Coefficients for Multivariate Normal Distributions,” Annals of Mathematical Statistics, Volume 22, 1951.

 

2. APPLICATIONS

Klein, L.R.: Economic Fluctuations in the United States, 1921-1941, Cowles Commission Monograph No. 11, New York, Wiley, 1950.

Schultz, Henry: The Theory and Measurement of Demand, Chicago, University of Chicago Press, 1938.

Wold, Herman: Demand Analysis; A Study in Econometrics, New York, Wiley, 1953.

 

3. SUGGESTED STATISTICAL REFERENCES

Mood, A.M., Introduction to the Theory of Statistics, New York, McGraw Hill, 1950.

Kendall, M.G.: The Advanced Theory of Statistics, 2 vols., London, Charles Griffin and Co., 1943, 1946.

Wilks, S.S.: Mathematical Statistics, Princeton, Princeton University Press, 1947.

 

Source: Harvard University Archives, Syllabi, course outlines and reading lists in Economics, 1895-2003, Box 6, Folder “Economics 1953-1954, (2 of 2)”.

Image Source: September 1961 entry card for John Somerset Chipman (b. 28 June 1926 in Montreal).  Rio de Janeiro, Brazil, immigration Cards, 1900-1965 at ancestry.com.

 

Categories
Columbia Economists Iowa Statistics

Columbia. Economics Ph.D. alumnus. BLS Commissioner, Royal Meeker, 1906

 

Having myself been an economics index number junkie for the better part of my career, I could naturally not resist creating this post for our Meet an Economics Ph.D. alumna(us) series. I first “met” Royal Meeker, the third Commissioner of the U.S. Bureau of Labor Statistics while identifying students who attended the advanced economics seminars conducted by John Bates Clark and Edwin R.A. Seligman at Columbia in 1900/01 and 1902/03. As you can see from his picture, he also provides a dapper addition to the Economists Wearing Bowties Collection.

_______________________

Royal Meeker
Commissioner of the Bureau of Labor Statistics
August 1913–June 1920
Appointed by: Woodrow Wilson

Royal Meeker was born in Susquehanna County, Pennsylvania in 1873. He attended college at Iowa State College, Columbia University, Seligman, and the University of Leipzig before becoming a professor of history, political science, and economics at Ursinus College in Pennsylvania. A year after publishing his dissertation in 1905, Meeker earned his Ph.D. from Columbia. When Meeker applied for and gained a position at Princeton in 1905, he made his first connection with Woodrow Wilson, then president of Princeton.

Wilson was elected President in 1912, and shortly afterward, Meeker offered to help by performing a survey of the economic community on the banking reform issue. Wilson found the information “most useful,” and, in June 1913, when Secretary of Labor Wilson recommended Meeker fill the position of Commissioner of Labor Statistics, President Wilson urged the Senate to accept. Meeker was sworn in on August 11.

A staunch believer in stressing the human factor in business, Meeker wanted, among other things, a nationwide system of public employment offices; workmen’s compensation; child labor restrictions combined with strong, State-controlled schools; and government action to protect workers. Meeker also sought to eliminate duplication of work by Government agencies, singling out six agencies competing with the Bureau.

During Meeker’s first years as Commissioner, he revised the index numbers of retail and wholesale prices, updated wage studies data collections, and began cost-of-living studies for the District of Columbia. In his concern for unemployment, Meeker ordered studies in 16 East and Middle West cities and 12 Rocky Mountain and Pacific Coast cities. The Bureau published the results in 1916 in the publication Unemployment in the United States. At the same time, the Bureau began a monthly series, “Amount of employment in certain industries,” which was the start of the Bureau’s establishment series on employment and total payrolls. In trying to reduce labor turnover by promoting improved working conditions in businesses, the Bureau surveyed corporate welfare plans from 430 employers.

In 1915, Meeker began supplementing the Bureau’s irregularly published bulletins with a new, monthly journal – the Monthly Review, now called the Monthly Labor Review. The journal expanded greatly, publicizing the first results of new Bureau surveys on cost of living, the new budget studies, and information on conditions in other countries. The Review later carried articles on the effect of war on wages, hours, working conditions, and prices in European countries.

Meeker also believed in creating national health insurance and safety programs. In 1916, he succeeded in convincing Congress to create a Board to administer the workmen’s compensation program, which had been under the Bureau’s responsibility since 1908. Working with a committee of the International Association of Industrial Accident Boards and Commissions, Meeker helped develop standard methods and definitions for reporting accidents. The Bureau offered to tabulate and publish State accident statistics, and in 1917, published Causes of Death by Occupation.

Meeker’s second term brought new challenges with the United States entering World War I. With the Government trying to adjust wages to rising costs of living, Meeker was permitted to create a comprehensive consumer expenditure survey. The Bureau began work by surveying the cost of living of families in shipbuilding, the results of which the Shipbuilding Board used to set uniform national wage rates for skilled shipyard trades.

Soon, the Bureau was allocated $300,000 to collect nationwide data on the cost of living. Conducted in 1918–19, the survey covered 12,000 families in 92 cities in 42 States. The results were published in the Monthly Labor Review in May 1919. Shortly thereafter, the Bureau issued its first comprehensive set of cost-of-living indexes for the Nation and for major industrial and shipbuilding centers. This marked the beginning of semiannual cost-of-living indexes for the Nation as a whole and for 31 cities.

To reflect wartime conditions and help resolve disputes, the Bureau was allotted $300,000 for an integrated study of occupational hours and earnings. The results, presented in May 1920, covered wages and hours during 1918 and 1919 for 780 occupations in 28 industries.

Meeker resigned in 1920 to head up the Scientific Division of the International Labor Office (ILO), a major office in the League of Nations. Secretary of Labor Wilson called Meeker “an exceptionally efficient administrator of the Bureau of Labor Statistics.” Secretary Wilson went on to describe Meeker’s three greatest accomplishments: coordinating the Bureau’s work with work performed by States and standardizing industrial terminology and methods; reorganizing the cost-of-living work on a family budget or market basket basis; and studying wartime wages and living costs that were accepted by all the wage boards.

After working for the ILO from 1920 to 1923, Meeker served as Secretary of Labor and Industry for the Commonwealth of Pennsylvania from 1923 to 1924. In 1924, he went to China as a member of the Commission on Social Research, and 1926–27, he taught economics at Carleton College in Minnesota. Meeker served as president of the Index Number Institute in New Haven from 1930 to 1936, and in 1941, he was named Administrative Assistant and Director of Research and Statistics of the Connecticut Department. He retired in 1946 and died in New Haven in 1953.

Source: United States. Bureau of Labor Statistics. Webpage: BLS History/Commissioners/Royal Meeker.

Image Source: Prof. Royal Meeker, U.S. Commissioner of Labor Statistics, 1914. Library of Congress Prints and Photographs Division Washington, D.C. 20540 USA

Categories
Chicago Economists Sociology Statistics

Chicago (1907-08). Economist turned Epidemiologist, Edgar Sydenstricker

The last name “Sydenstricker” is certainly not all-too-common which is probably a reason that it lodged in my memory after I transcribed the 25th anniversary of the University of Chicago’s Department of Political Economy. Elgar Sydenstricker was included there in the list of “Fellows of Political Economy”. Nonetheless, I had no record of him ever completing a Ph.D. there (he never did).

With the coming of the Covid-19 pandemic, I thought it might be worth a look to see which economists (if any), were involved in the scientific analysis of the influenza epidemic of 1918-19. The name “Edgar Sydenstricker” was everywhere. And yes, it was the University of Chicago ABD, Edgar Sydenstricker.

I realized there was a significant gap in my rather exclusive focus on Ph.D. academic economists. Someone like Edgar Sydenstricker had an academic economist’s training, but he was not part of the self-perpetuating caste of economics professors.

With the influenza epidemic of 1918-19, Edgar Sydenstricker became a leading statistician in the efforts to advance epidemiology.  Today’s post gives information about his career and publications.

Fun fact: his younger sister was Pearl Sydenstricker Buck (1938 Nobel Prize in literature).

______________________

Best single source about Edgar Sydenstricker
(includes a bibliography)

Kasius R.V., ed. The challenge of facts. Selected public health papers of Edgar Sydenstricker. New York: Prodist, for the Milbank Memorial Fund, 1974.

Wiehl, D.G. Edgar Sydenstricker: a memoir. pp. 1-17.

______________________

Edgar Sydenstricker’s Time-line.
(b. July 15, 1881 in Shanghai; d. Mar 19, 1936 in New York City).

Parents were missionaries from West Virginia, Rev. Dr. Absalom and Caroline Stulting Sydenstricker.

1896. Edgar Sydenstricker came to United States

1900. A.B., Fredericksburg College (Virginia).

1902. M.A. (honors) in sociology and economics at Washington and Lee.

1902-1905. High school principal in Onancock, Virginia

1905.  Editor of the Daily Advance in Lynchburg, Virginia

1907-08. Graduate study at University of Chicago [fellow in political economy]

1908-1915. United States Immigration Commission and Commission on Industrial Relations. Extensive surveys of wages, working conditions, and scales of living of industrial workers, especially in industries with large numbers of foreign born.

1915. Joins United States Public Health Service as first statistician ever. He was hired to assist Dr. B. S. Warren [studied health and economic status of garment workers in New York City, sickness insurance in Europe].

1916-20. Sydenstricker and Joseph Goldberger studied causes of pellagra in the American South.

1917. Elected member of the American Statistical Association.

1918. With Wade Hampton Frost research on statistics of influenza [papers by Sydenstricker, Wade Hampton Frost, Selwyn D. Collins, Rolo H. Britten and others at the Public Health Service giving “a most comprehensive history of influenza from 1910 to 1930”].

1920. Appointed head of Office of Statistical Investigations.

1921. Begins Hagerstown Morbidity Survey [which later became the U.S. National Health ].

1922. Becomes fellow of the American Statistical Association

1923. League of Nations invited him to establish the Epidemiological Service of the Health Organization.

1925. Consultant to Milbank Memorial Fund

1928. Director of research of Milbank Memorial Fund.

1931-34. Represented ASA at Social Science Research Council.

1935. Scientific director of Milbank Memorial Fund

1936, March 19. Died of cerebral hemorrhage.

______________________

The important influenza studies of the Public Health Reports, U.S.

United States Treasury Department and the Public Health Service. Influenza Morbidity and Mortality Studies, 1910-1935. Reprints from the Public Health Reports. Washington: USGPO, 1938.

Influenza-pneumonia mortality in a group of about 95 cities in the United States, 1920-29. By Selwyn D. Collins. Reprint 1355, from Public Health Reports, Vol. 45, No. 8 (February 21, 1930), pp. 361-406.

Influenza and pneumonia mortality in a group of about 95 cities in the United States during four minor epidemics, 1930-35, with a summary for 1920-35. By Selwyn D. Collins and Mary Gover. Reprint 1720, from Public Health Reports, Vol. 50, No. 48 (November 29, 1935), pp. 1668-1689.

Mortality from influenza and pneumonia in 50 large cities of the United States, 1910-29. By Selwyn D. Collins, W. H. Frost, Mary Gover, and Edgar Sydenstricker. Reprint 1415, from Public Health Reports, Vol. 45, No. 39 (September 26, 1930), pp. 2277-2328.

Excess mortality from causes other than influenza and pneumonia during influenza epidemics. By Selwyn D. Collins. Reprint 1553, from Public Health Reports, Vol. 47, No. 46 (November 11, 1932), pp. 2159-2179.

The incidence of influenza among persons of different economic status during the epidemic of 1918. By Edgar Sydenstricker. Reprint 1444, from Public Health Reports, Vol. 46, No. 4 (January 23, 1931), pp. 154-170.

Age and sex incidence of influenza and pneumonia morbidity and mortality in the epidemic of 1928-29 with comparative data for the epidemic of 1918-19. By Selwyn D. Collins. Reprint 1500, from Public Health Reports, Vol. 46, No. 33 (August 14, 1931), pp. 1909-1937.

The influenza epidemic of 1928-29 in 14 surveyed localities in the United States. By Selwyn D. Collins. Reprint 1606, from Public Health Reports, Vol. 49, No. 1 (January 5, 1934), pp. 1-42.

______________________

Other Sydenstricker articles on public health

Edgar Sydenstricker. Existing Agencies for Health Insurance in the United States,” in U.S. Department of Labor, Proceedings of the Conference on Social Insurance, 1916 (Washington, D.C.: U. S. Government Printing Office, 1917), pp. 430-75.

Edgar Sydenstricker. Preliminary Statistics of the Influenza Epidemic, in Epidemic Influenza. Prevalence in the United States. Public Health Reports. Vol. 33, No. 52 ( December 27, 1918), pp. 2305-2321.

Sydenstricker, E., King W.I.A. A method for classifying families according to incomes in studies of disease prevalence. Public Health Reports 1920; 35: 2828-2846.

Sydenstricker, E. Health and Environment. New York: McGraw-Hill, 1933.

Sydenstricker, E. Health and the Depression. Milbank Memorial Fund Q 1934; 12:273-280.

Sydenstricker, E. The incidence of illness in a general population group: General results of a morbidity study from December 1, 1921 through March 31, 1924 in Hagerstown, Md. Public Health Reports. 1925; 40: 279-291.

Milbank Memorial Fund. Program of the Division of Research 1928-1940. (1941)

 

Image Source:  Portrait of Edgar Sydenstricker in Washington and Lee University Yearbook The Calyx, 1902.

Categories
Columbia Economists Statistics

Columbia. Promotions and Memorial Minute for Abraham Wald

 

 

Abraham Wald (1902-1950) got his foot into the Columbia economics department door thanks to a grant from the Carnegie Foundation arranged for him by Harold Hotelling in 1938. In this post we follow Wald’s Columbia career up through the faculty memorial minute that followed his tragic, untimely death in an airplane accident during a lecture tour in India in December 1950.

___________________

Promotion to Assistant Professor

From the November 26, 1941 letter to President Nicholas Murray Butler from Robert M. Haig, Chairman of the department of economics (pp. 3-5 and supporting annex B).

“…We feel that it is important, if at all possible, that the following action be taken.

  1. Appoint Abraham Wald to an assistant professorship at $3,600 (Wald is now a lecturer at $3,000, of which $2,400 is financed by a special grant, the continuance of which is not assured.
    (See Annex B)

A recent development in the case of Wald is an offer of a permanent post (presumably an assistant professorship) at Queens College. This post will be open to him in case it proves impossible for us to give him the status recommended. Our enthusiasm for him has increased since last year when I wrote as follows:—

“The position of this recommendation at the very head of our list is attributable primarily to a conviction that Abraham Wald is an unusually interesting gamble. By risking a moderate stake, the University can put itself in a position where it may (and in our judgment probably will) be rewarded a hundred-fold. For Wald is not only a young scholar whose attainments are of a high order of merit, but one whose potentialities are obviously large.

“When Wald came to us two years ago, as a lecturer whose stipend was supplied by a special and presumably temporary grant, we were frankly apprehensive lest we should presently find ourselves indirectly committed, without adequate consideration, to a permanent addition to our staff concerning whom we might not be enthusiastic. Consequently care was exercised to make it clear to all concerned that his appointment as a lecturer supported by a special grant carried with it no future obligation. Fortunately then, we are able to approach the consideration of his case at this juncture free from any pressure of old commitments, express or implied.

“Our recommendation of Wald should be interpreted then as a free and fresh expression of our admiration for his accomplishments and of our faith in his future. As a result of our contacts with him and with his work, we are convinced that here is a man whose contributions are reasonably certain to continue to break new ground on a section of the frontier of knowledge where notable progress seems imminent.

“We recognize that Wald’s field is one that is of interest and significance to several departments of the University and that there are unsettled questions as to whether ultimately it should be attached to our own or to some other department or whether it should constitute a separate department in its own right. Irrespective of the answers that may ultimately be given to such questions of structural organization, we, in the Department of Economics, desire to express the hope that it will prove possible for the University to provide for the further development of teaching and research in statistics on a high level and we wish to take this opportunity to make it clear that, pending a final decision as to organization, we should consider it an honor to be permitted to shelter and to stand sponsor for scientific work such as that of Wald.”

[…]

ANNEX B.
BIOGRAPHIC MEMORANDUM OF ABRAHAM WALD

I was born in Cluj, October 31, 1902. I studied at the University in Cluj and at the University of Vienna, and obtained my doctor’s degree in mathematics at the University of Vienna in 1930. The subject of my doctor’s thesis was the Hilbert system of axioms of Geometry.

For the next four years I did mathematical research at the University of Vienna and collaborated with Professor Karl Menger. I was co-editor with him of “Ergebnisse eines mathematischen Kolloquiums.” During this time my interests were chiefly in general abstract and metric geometry, theory of probability, and mathematical economics, in which fields my papers were written.

In 1934 I became in addition a research associate of the Institute for Business Cycle Research in Vienna and published several papers in mathematical economics. My interest in statistics and its application in economics dates from this time, when I became the statistical expert of the Institute.

After the annexation of Austria, I came to the United States and was for several months a fellow of the Cowles Commission for Research in Economics. Thereafter I became a lecturer at Columbia University which is the position I hold at present. Since my arrival in the United States I have been interested chiefly in statistics and mathematical economics and have published a series of papers in these fields. I have been elected a fellow of both the Institute of Mathematical Statistics and the Econometric Society.

LIST OF PUBLICATIONS

  1. Abstract and metric geometry
    1. Über den allgemeinen Raumbegriff, Ergebnisse eines math. Kolloquiums, Heft 3, Vienna. [1931]
    2. Axiomatik des Zwischenbegriffes in metrischen Räumen, Mathematische Annalen, Vol. 104. [1931]
    3. Der komplexe euklidische Raum [Komplexe und indefinite Räume], Erg. eines mathem. Kolloquiums, Heft 5, Vienna.
    4. Indefinite euklidischen Räume, Erg. eines mathem. Kolloquiums, Heft 5, Vienna.
    5. Vereinfachter Beweis des Steinitzschen Satzes, Erg. mathem. Kolloquiums, Heft 5, Vienna.
    6. Bedingt konvergente Reihen von Vektoren, Erg. mathem. Kolloquiums, Heft 5, Vienna.
    7. Riehen in topologischen Gruppen, Erg. mathem. Kolloquiums, Heft 5, Vienna.
    8. Eine neue Definition der Flächenkrümmung, Erg. mathem. Kolloquiums, Heft 6, Vienna
    9. Sur la courbure des surfaces, C. R. [Acad. Sci.] Paris, 1935.
    10. Aufbau [Begründung] einer kooridinatenlosen Differentialgeometrie der Flächen, Erg. mathem. Kolloquiums, Heft 7, Vienna.
    11. Eine Characterisierung des Lebesgueschen Masses, Erg. mathem. Kolloquiums, Heft 7, Vienna.
  1. Probability, Statistics and Mathematical Economics.
    1. Sur la notion de collectif dans le calcul des probabilités, C.R. [Acad. Sci.] Paris, 1936.
    2. Die Widerspruchsfreiheit des Kollektivbegriffes, Erg. mathem. Kolloquiums, Heft 8, Vienna.
    3. Die Widerspruchsfreiheit des Kollektivbegriffes, Actualités Scientifiques, 1938, Paris.
    4. Berechnung und Ausschaltung von Saisonschwankungen, Julius Springer, Vienna, 1936.
    5. Zur Theorie der Preis Indexziffern, Zeitschrift für Nationalökonomie, Vienna, 1937.
    6. Über die Produktionsgleichungen der ökonomischen Wertlehre, Erg. mathem. Kolloquiums, Heft 6, Vienna.
    7. Über die Produktionsgleichungen der ökonomischen Wertlehre, zweite Mitteilung, Erg. mathem. Kolloquiums, Heft 7, Vienna.
    8. Über einige Gleichungssysteme der mathematischen Ökonomie, Zeitschrift für Nationalökonomie, Vienna, 1936. [translated into English by Otto Eckstein, Econometrica, 1951, pp. 368-403]
    9. Extrapolation des gleitenden 12-Monatsdurchschnittes, Beilage zu den Berichten des Öster. Institutes für Konjunkturforschung, Vienna, 1937.
    10. Grundsaetzliches zur Berechnung des Produktionsindex, Beilage zu den Berichten des Öster. Institutes für Konjunkturforschung, Vienna, 1937.
    11. Generalization of the inequality of Markoff, The Annals of Math. Statistics, December, 1938.
    12. Long Cycles as a result of repeated integration, American Mathem. Monthly, March, 1939.
    13. Confidence limit for continuous distribution functions (co-author J. Wolfowitz), The Annals of Math. Statistics, June, 1939.
    14. Limits of a distribution function determined by absolute moments, Transact. of the Amer. Mathem. Society, September, 1939.
    15. A new formula for the index of cost of living, Econometrica, October, 1939.
    16. Contributions to the theory of statistical estimation, The Annals of Mathem. Statistics, December, 1939.
    17. A note on the analysis of variance with unequal class frequencies, The Annals of Mathem. Statistics, March, 1940.
    18. The approximate determination of indifference surfaces, Econometrica, April, 1940.
    19. On a test whether two samples are from the same population (with J. Wolfowitz), The Annals of Mathem. Statistics, June, 1940.
    20. Fitting of straight lines when both variables are subject to error, The Annals of Mathem. Statistics, September, 1940.
    21. Asymptotically most powerful tests of statistical hypotheses, Annals of Mathem. Statistics, March, 1941.
    22. Some examples of asymptotically most powerful tests will appear in the December, 1941 issue of the Annals of Mathem. Statistics.
    23. Asymptotically shortest confidence intervals paper presented at the meeting of the Amer. Math. Soc., September, 1940. Accepted for publication in the Annals of Mathem. Statistics.
    24. On the distribution of Wilks’ statistic for testing the independence of several groups of variates (in collaboration with R. Brookner), Annals of Mathem. Statistics, June, 1941.
    25. The large sample distribution of the likelihood ratio statistics, paper presented at the meeting of the Institute of Mathem. Statistics, September, 1941, Chicago. It will be published in the Annals of Mathem. Statistics.
    26. On testing statistical hypotheses concerning several unknown parameters, paper presented at the meeting of Amer. Mathem. Society, February, 1941, New York City. It will be published in the Annals of Mathem. Statistics.
    27. On the analysis of variance in case of multiple classifications with unequal class frequencies, Annals of Mathem. Statistics, September, 1941.

Source: Columbia University Archives. Central Files 1890—Box 386, Folder “Haig, Robert Murray 7/1941-6/1942”

Cf: “The Publications of Abraham Wald” [1931-1952] was published in The Annals of Mathematical Statistics 23:1 (March 1952, pp. 29-33.

___________________

Aliens in the Department of Economics

December 19, 1941

Mr. Philip M. Hayden, Secretary,
213 Low Memorial Library.

Dear Mr. Hayden:

In reply to the request contained in your recent Memorandum to executive officers, I report the following aliens from the Department of Economics:

Harold Barger

29 West 8th Street, New York City

Nationality: British
Age: 34
Alien Registration No.: 3239174

 

Donald Bailey Marsh

106 Morningside Drive, New York City

Nationality: Canadian
Age: 30
Alien Registration No.: 1152252

 

Robert Valeur

40 Barrow Street, New York City

Nationality: French
Age: 38
Alien Registration No.: 5061531

 

Abraham Wald

241 West 108th Street, New York City

Nationality: born in Kolozsvar [Note: Hungarian spelling of Cluj-Napoca], Transylvania, Alien Registration officials were in doubt how to describe nationality.
Age: 39
Alien Registration No.: 4506027

Very truly yours,
Chairman, Department of Economics

Source: Columbia University Libraries, Manuscript Collections. Department of Economics Collections, Faculty. Box 2, Folder “Faculty Beginning Jan 1, 1944 [sic]”.

___________________

Promotion to Associate Professor

February 7, 1944

Professor Abraham Wald,
608 Fayerweather.

Dear Professor Wald:

I am authorized by the Provost of the University to inform you that in the provisional budget for the academic year 1944-45 you are designated associate professor of Statistics at an annual stipend of $5,000. This provisional budget goes to the Trustees with the approval of the Committee on Educational Policy. While your promotion is not final until it is adopted by the Trustees at their meeting on the first Monday in April, the Provost and I agree that there is no reason whatsoever to doubt that the recommendation for your advancement will be approved, and that you run hardly an risk in declining the offer of an associate professorship at the University of Chicago.

As an associate professor you would hold your position at the pleasure of the Trustees, i.e., you would no longer be subject to year-to-year appointment and would, in effect, have continuing tenure. The position of associate professor in this respect is the same as that of a full professor.

I should like to add my personal assurance that the Department and the Administration stand behind the recommendation for your advancement. The reputation that you have won for yourself at Columbia is a very high one indeed. You have the friendship and warm support of all of your associates in the graduate faculties. I believe that you will have here a rich and promising career of creative scholarship.

Sincerely yours,

[copy unsigned, Frederick C. Mills?]

Source Columbia University Libraries, Manuscript Collections. Columbiana. Department of Economics Collection. Box 3 Budget, 1915-1946-47, Folder “Budget Material 1944-1945”.

___________________

Promotion to Professor

April 23, 1945

President Nicholas Murray Butler,
Columbia University.

Dear Mr. President:

A recent development makes it necessary for me to supplement my letter of November 30th, 1944, in which I submitted to you a provisional budget for the Department of Economics for the year 1945-46. Professor Abraham Wald, who occupies a place of strategic importance in our work in Mathematical Statistics, has received a very attractive offer from another institution. If we are to hold him at Columbia we must give him some advancement here. Although I am reluctant to approach you at this time, to request that you re-open the Department budget for next year, it is my strong opinion that this should be done. This opinion is shared by my colleagues who are interested in Columbia’s past and prospective accomplishments in Mathematical Statistics.

Work in Mathematical Statistics in American universities is in a pioneering stage. The fundamental bases of statistics, in mathematics and logic, have recently been materially extended. New horizons have been opened. We may expect in the next several decades further fruitful advances bearing upon the whole range of inquiry in the social and the natural sciences and in the arts of production and administration. In this field Columbia has already, through the work of Hotelling and Wald, achieved a leading position, one that is recognized throughout the world. Some indication of Columbia’s standing, and of the scientific and practical fruitfulness of our work in this field, I given by the accomplishments of the Statistical Research Group now serving the Army and the Navy as part of Columbia’s contribution to the war effort.

Columbia must hold and extend the position of preeminence we have won. We believe that in Hotelling and Wald we have men of intellectual vigor and established scientific competence who will be in the forefront of future advances in Mathematical Statistics. Their work will supplement and strengthen that of the Watson Scientific Computing Laboratory, in which Columbia will cooperate with the International Business Machines Corporation, as the work of that Laboratory will enhance the effectiveness of our efforts in Mathematical Statistics.

The scholarly record of Professor Wald is set forth in an attached statement. I need only add here that Wald’s researches in statistical theory have been fundamental in character and seminal in their influence. A recent outstanding example of the fertility of his thought is provided by his contribution of a new mathematical basis for techniques of quality control in manufacturing production, techniques that have been widely adopted in the control of war production. The sequential methods utilized in Dr. Wald’s procedures are capable of application in scientific experiments, and in a wide variety of other fields.

In the conviction that Columbia should reinforce success, in planning for the future, and should build where firm foundations have already been laid, I urge that the position of the University in the field of Mathematical Statistics be maintained, and strengthened. Dr. Abraham Wald’s continuance here is crucial in such a program. I recommend, therefore, that Dr. Wald, who is now Associate Professor of Statistics at an annual salary of $5,000, be appointed Professor of Mathematical Statistics, at a salary of $7,500 a year, the appointment to be effective July 1, 1945.

Respectfully submitted,

FREDERICK C. MILLS

Source Columbia University Libraries, Manuscript Collections. Columbiana. Department of Economics Collection. Box 3 Budget, 1915-1946-47, Folder “Budget Material 1944-1945”.

___________________

April 27, 1951

Memorial Minute for Professor A. Wald

Professor Wolfowitz then presented a minute memorializing the late Professor Abraham Wald. It was adopted by a rising vote, and a copy was ordered sent to Professor Wald’s family.

ABRAHAM WALD

Abraham Wald, Professor of Mathematical Statistics and a distinguished member of this Faculty, was killed in an airplane accident in India on December 13, 1950. He had been on a lecture tour of Indian universities and research centers. Mrs. Wald was killed in the same accident.

Dr. Wald arrived in the United States in the summer of 1938, a refugee from Nazi persecution. In the fall of 1938 he came to Columbia as a fellow of the Carnegie Corporation. He became a member of this Faculty in 1942 and professor of Mathematical Statistics in 1945. Much of his statistical work was done here. It shed luster on Columbia and largely helped to make Columbia an important center of mathematical statistics. His work changed the whole course and emphasis of modern mathematical statistics. In addition to many other contributions the theory of statistical decision functions and the theory of sequential analysis were founded by him. He also made important contributions to mathematical economics, the theory of probability, and metric geometry.

He was a good friend to many, a genial colleague, and an inspiring teacher. By his death the University and science have sustained a grievous loss.

Source: Columbia University Archives. Minutes of the Faculty of Political Science, 1950-1962.

Image Source: Naval Ordnance Test Station, Inyokern, California from the obituary by J. Wolfowitz published in The Annals of Mathematical Statistics 23:1 (March, 1952), pp. 1-13.

 

Categories
Columbia Economists Pennsylvania Statistics

Columbia. Statistics PhD alumnus. Robert E. Chaddock, 1908

 

The post provides another life/career overview of a Ph.D. alum. Today’s Ph.D. went on to become professor of sociology and statistics at Columbia University, Robert Emmet Chaddock.

__________________________

Previous posts at Economics in the Rear-view Mirror
with Chaddock content…

Request for funding for his statistical laboratory in 1911 (with a newspaper account of his 1940 suicide).

E.R.A. Seligman’s recommendation for Chaddock’s promotion to Associate Professor in 1912.

__________________________

Chaddock obituaries by…

Frederick E. Croxton in Journal of the American Statistical Association 36:213 (March, 1941), pp. 116-119.

William F. Ogburn in American Journal of Sociology 46:4 (January, 1941), p. 595.

__________________________

Robert E. Chaddock (1879-1940)

1879 born April 16, in Minerva, Ohio

1900 A.B. Wooster College

1900-05. Taught at Wooster College

1906 M.A., Columbia University

1906-08. University Fellow in Sociology, Columbia University

1908 Worked with the boy’s club of the Union Settlement (New York City)

1907-09. Instructor, Columbia University

1908 Ph.D. Columbia University.

1909-11. Assistant Professor of Economics and Statistics. Wharton School of the University of Pennsylvania.

1911-12. Assistant Professor of Statistics, Columbia University

1912-22. Associate Professor of Statistics, Columbia University

1917-24. Secretary-Treasurer of the American Statistical Association

1925 Publication of Principles and Methods of Statistics.

1922-40. Professor of Sociology and Statistics.

1925 President of the American Statistical Association

1925-1940. Member of the Joint Advisory Committee to the Director of the Census.

1928 Represented the Social Science Research Council as delegate to the International Conference on Population in Paris (July).

1929 LL.D. awarded by Wooster College

1933-36. Member of the Committee on Government Statistics and Informational Services (jointly established by the American Statistical Association and the Social Science Research Council)

1937 Chairman of the Joint Advisory Committee to the Director of the Census

1940 October 21. Death by suicide.

Other memberships

Member of the American Committee of the International Union for the Scientific Study of Population Problems.

Chairman of the Research Committee, member of the Executive Committee of the Research Bureau of the Welfare Council of New York City.

Consultant statistician of the Commonwealth Fund

Member of the Advisory Council of the Milbank memorial Fund.

Member and Vice-Chairman of the Committee on Research in Medical Economics. Member of the editorial board of the quarterly journal Medical Care.

Fellow of the American Public Health Association

Member of the International Statistical Institute

Member of the American Sociological Society

Member of the Century Club (New York)

Phi Beta Kappa.

__________________________

Political Science Faculty Memorial Minute
Columbia University

Dec. 13, 1940

Robert Emmet Chaddock

Professor Robert Emmet Chaddock served his University for over thirty years. Born at Minerva, Ohio, 1879, he took his A.B. at Wooster College in 1900. From the time he first came to Columbia as a graduate student in 1905, his association with the University was broken only for two years, during which he was Assistant Professor of Economics and Statistics at the Wharton School of the University of Pennsylvania. At Columbia he was in turn University Fellow in Sociology, Instructor in Economics, Assistant Professor of Statistics, Associate Professor, and from 1922 until his death Professor of Sociology and Statistics. In these various capacities he fulfilled his duties with unsparing devotion, giving attention to his students, whole-heartedly cooperating with his colleagues, freely participating in various organizations for the advancement of public welfare, and contributing always to the improvement of statistical application to social problems, especially those connected with population and public health. He was the author of numerous articles and reports on these and other subjects, and his work on Principles and Methods of Statistics has given guidance to a large number of students throughout the country. His recognition as a leader in this field was shown by the many calls made upon his services, from the Bureau of the Census, the Milbank Foundation, the Welfare Council of New York City, the Cities Census Committee, and the International Statistical Institute, and other bodies. To these calls Professor Chaddock never failed to respond. He won the regard of all who knew him. His death removes a man who gave himself without limit and without afterthought, to his University, to his family, to the community. His colleagues tender their respectful sympathy to those who intimately mourn for him, his wife and daughter.

Robert M. MacIver
Carlton J. H. Hayes
Roswell C. McCrea

 

Source: Columbia University Archives. Minutes of the Faculty of Political Science, 1940-1949, p. 881.

Image Source: Robert Emmet Chaddock from Barnard CollegeMortarboard, 1919.

 

Categories
Chicago Economists Harvard Statistics

Harvard. Semester exams for Statistics. John Cummings, 1896-1900

 

 

 

John Cummings was awarded the first Ph.D. in political economy at the University of Chicago in 1894. His doctoral thesis was “The Poor Law system of the United States”, later published as “The Poor Laws of Massachusetts and New York.” Publications of the American Economic Association, vol. X, no. 4 (July, 1895). His first real academic job was at Harvard, after which he went to have a successful career as a statistician in government service. He was apparently quite a big name in vocational education policy by the end of his career.

This post provides the questions to all of the semester exams from the times he taught the statistics course for when he taught at his undergraduate alma mater during the last five years of the 19th century.

Life and Career of John Cummings

1868. Born May 18 in Colebrook, New Hampshire.

1887. Entered Harvard College.

1891. A.B., magna cum laude, Harvard College

1892. A.M., Harvard College

1893-94. Senior Fellow, Department of Political Economy, University of Chicago.

1894. Ph.D. in Political Economy; Reader in Political Economy, University of Chicago.

1894-1900. Instructor in Economics, Harvard University.

1900-02. Editorial staff New York Evening Post.

1902. Married Carrie R. Howe in Marion, Indiana, December 3, 1902)

1902-10. Assistant Professor in Political Economy. University of Chicago.

1910-16. Expert special agent, Census Bureau.

1917-23. Statistician, Federal Board for Vocational Education, Washington, D.C.

1924-30. Statistician and economist, Division of Research and Statistics, Federal Reserve Board.

1930-1933. Chief of Research and Statistics, Federal Board for Vocational Education, Washington, D.C.

1933-. Chief of research and statistical service, vocation education, United States Office of Education.

1936. Died , June 26.  in Washington, D.C.

Buried at the Crown Hill Cemetery, Indianapolis, Indiana.

Sources: Obituary in Washington Post, June 27, 1936, p. 8. Also “A Tribute to Dr. Cummings” in School Life (September 1936), p. 12.

_____________________

Tribute to Memory of John Cummings

At the annual meeting of the National Committee on Research in Secondary Education, of which organization Dr. Cummings was a member, the following resolution was adopted honoring his memory:

In the passing of Dr. John Cummings, of the United States Office of Education, research lost one of its most careful and effective workers. For a period of more than 20 years, Dr. Cummings was in the forefront of development in vocational education throughout the Nation. As research expert for the Joint Congressional Committee on National Grants for Education during President Wilson’s administration, he was instrumental in providing the bases upon which the legislation known as the Smith-Hughes Act was developed. Subsequently, as Chief of the Research and Statistical Service of the Vocational Educational Division in the Federal Office of Education, he was identified closely with the expansion and improvement of services in his field of work.

Dr. Cummings had the confidence and respect of his associates. By disposition he was kindly, tolerant, and friendly. He was never too busy to help those who came to him for counsel and advice. Gentle and reserved, he was at the same time an aggressive champion of objectives and principles in which he believed. His was a brilliant mind and an indomitable spirit. The National Committee on Research in Secondary Education can pay him no better or more deserved tribute than that voiced by his chief, Dr. J. C. Wright. Assistant Commissioner for Vocational Education, when he said: “As an economist, statistician, and editor, Dr. Cummings rendered invaluable service to the cause of vocational education in the United States. He was a man of outstanding ability, brilliant mentality, and quiet, unassuming personality. The Office of Education, and more particularly the cause of vocational education, has suffered a distinct loss in his death.”

SourceSchool Life, vol. 22 (April, 1937), p. 236.

_____________________

Harvard Course: Statistics

Course Description
(1897-98)

[Economics] 4. Statistics. — Applications to Social and Economic Problems. — Studies in the Movements of Population. — Theory and Method. Mon., Wed., Fri., at 11. Dr. John Cummings.

This course deals with statistical methods used in the observation and analysis of social conditions, with the purpose of showing the relation of statistical studies to Economics and Sociology, and the scope of statistical inductions. It undertakes an examination of the views entertained by various writers regarding the theory and use of statistics, and an historical and descriptive examination of the practical methods of carrying out statistical investigations. The application of statistical methods is illustrated by studies in political, fiscal, and vital statistics, in the increase and migration of population, the growth of cities, the care of criminals and paupers, the accumulation of capital, and the production and distribution of wealth.

Source: Harvard University. Faculty of Arts and Sciences. Division of History and Political Science Comprising the Departments of History and Government and Economics, 1897-98, p. 37.

_____________________

Course Enrollment 1895-96
(Half-course)

[Economics] 42. Dr. John Cummings. — Theory of Statistics. — Applications to Social and Economic Problems. — Studies in movements of population. Hf. 3 hours. 2d half-year

Total 19: 2 Graduates, 11 Seniors, 4 Juniors, 2 Sophomores.

Source: Harvard University. Report of the President of Harvard College, 1895-1896, p. 64.

 

1895-96.
ECONOMICS 4.
Year-End Final Examination.

[Divide your time equally between A. and B.]

A.
I and II may be treated as one question.

  1. What do you understand by “movement of population”? What light do Statistics throw upon the law of population as stated by Malthus?
  2. What are some of the “more striking facts and more pregnant results of the vast growth of population in Europe, America, and the British Colonies within the last half century”?

 

B.
Take five.

  1. In constructing a life table what correction must be made for abnormal age and sex distributions of the population?
  2. Define the following terms: “Mortality,” “Expectation of Life,” “Mean Duration of Life.” How should you calculate the mean duration of life from the census returns for any community?
  3. How should you calculate the economic value of a population?
  4. What are some of the inaccuracies to which censes enumerations are liable?
  5. What is the nature of a statistical law? Of what categories of social phenomena may statistical laws be formulated? In what sense are they laws? How do they bear upon freedom of the will in human conduct?
  6. How do the conditions of observation in social sciences differ from conditions of observation in the natural sciences?
  7. What do you understand by the law of criminal saturation?
  8. By what considerations should the Statistician be guided in making selection of social phenomena for investigation?

Source: Harvard University Archives. Harvard University, Examination papers, 1873-1915. Box 4, Bound volume: Examination Papers 1896-97. Papers set for Final Examinations in History, Government, Economics, Philosophy, Fine Arts, Architecture, and Music in Harvard College. June 1896, pp.38-39.

_____________________

Course Enrollment 1896-97
(Year-course)

[Economics] 4. Dr. John Cummings. — Theory and Methods of Statistics. — Applications to Economic and Social Questions. — Studies in the Movement of Population. 3 hours.

Total 15: 8 Seniors, 7 Juniors.

Source: Harvard University. Report of the President of Harvard College, 1896-1897, p. 65.

 

1896-97.
ECONOMICS 4.
Mid-Year Examination.

[Divide your time equally between A. and B.]

A.

  1. The development of scientific statistics and the statistical method as employed in the social sciences.
  2. Social and economic causes of the migratory movements which have taken place in the populations of Europe and America during this century, and the laws in accordance with which those migrations have taken place where you can formulate any.

B.
(Take five.)

  1. Rural depopulation and the growth of cities in the United States.
  2. Define: “mean after life,” “expectation of life,” “mean duration of life,” “mean age at death.” What relation does the mean age of those living bear to the mean age at death? To the mean duration of life?
  3. Anthropological tests of race vitality as applied to the American negro?
  4. Explain how the economic value of a population is effected by its age and sex distribution.
  5. The United States census: either (1) an historical account of it, or (2) an account of the work now undertaken by the Census Bureau.
  6. Explain the various methods of calculating the birth rate of a population.
  7. How far are social conditions in a community revealed in the birth rate, the death rate, the marriage rate? Of what are fluctuations in these rates evidence in each case?
  8. What do you understand by the “index of mortality”?

Source: Harvard University Archives. Harvard University Mid-year Examinations, 1852-1943. Box 4. Bound volume: Examination Papers, Mid-Years. 1896-97.

 

1896-97.
ECONOMICS 4.
Year-End Final Examination.

I.

  1. Give an historical account of the United States census, and a general statement of the ground covered in the census of 1890; also show how the census taking is supplemented by work done in the Department of Labor and in the statistical bureaus established in connection with the several administrative departments.
  2. Define Körösi’s “rate of natality,” and state any statistical evidence you know that the rate is affected by the standard of living.
  3. “It must, at all times, be a matter of great interest and utility to ascertain the means by which any community has attained to eminence among nations. To inquire into the progress of circumstances which have given pre-eminence to one’s own country would almost seem to be a duty….The task here pointed out has usually been left to be executed by the historian.” Porter: “The Progress of the Nation.”
    What contribution has statistics to make in the execution of this task? What do you understand to be the nature of the statistical method, and what are the legitimate objects of statistical inquiry?

II.
[Take two.]

  1. What light does statistics throw upon the “natural history of the criminal man”?
    Give Ferri’s classification of the “natural causes” of crime, and comment upon that classification. Of criminals.
    What do you understand by “rate of criminality”? By “criminal saturation”?
  2. To what extent in your opinion is suicide an evidence of degeneration in the family stock?
    Discuss the influence upon the rate of suicide of education, religious creed, race, climate and other facts of physical, political and social environment.
  3. Comment critically upon the tables relating to crime in the last five federal censuses taken in the United States.
  4. What difficulties beset a comparative study of criminality in different countries?
  5. How far is it possible to give a quantitative statement to moral and social facts?

III.
[Take one.]

  1. What are some of the more salient facts concerning the movement of population and wealth in the United States, England, and France during the present century, so far as those facts are evidenced in the production, consumption and distribution of wealth?
  2. Discuss the movement of wages and prices in the United States since 1890.
  3. What do you understand by “index figures,” “average wages,” “average prices,” and “weighted averages”?

IV.
[Take one.]

  1. How do you account for the increase in the proportion of urban to rural population during this century? What statistical evidence is there that the increased density of a population affects the mean duration of life? What importance to you attach to this evidence?
    Explain the effect of migratory movements upon the distribution of a population according to age, sex and conjugal condition, and upon the birth rate, death rate and marriage rate.
  2. Define and distinguish: “mean age at death”; “mean duration of life”; mean age of those living”; expectation of life.”
  3. The “law of population” as formulated by Malthus and by subsequent writers.

Source: Harvard University Archives. Harvard University, Examination papers, 1873-1915. Box 4, Bound volume: Examination Papers 1896-97. Papers set for Final Examinations in History, Government, Economics, Philosophy, Fine Arts, Architecture, and Music in Harvard College. June 1897, pp. 39-41.

_____________________

Course Enrollment 1897-98
(Year-course)

[Economics] 4. Dr. J. Cummings. — Statistics. — Applications to Economic and Social Questions. — Studies in the Movement of Population. — Theory and Method. 3 hours.

Total 18: 7 Seniors, 7 Juniors, 3 Sophomores, 1 Other.

Source: Harvard University. Report of the President of Harvard College, 1897-1898, p. 78.

 

1897-98.
ECONOMICS 4.
Mid-Year Examination.

[Divide your time equally between A. and B.]

A.
[Take two.]

  1. In what sense do you understand Quetelet’s assertion that “the budget of crime is an annual taxation paid with more preciseness than any other”?
    Comment upon the “element of fixity in criminal sociology.”
    What are the “three factors of crime”?
    Can you account for the “steadiness of the graver forms of crime”? for the increase or decrease of other crimes?
    Define “penal substitutes.”
    What determines the rate of criminality?
    Comment upon the tables relating to crime in the last federal census, and explain how far they enable one to estimate the amount of crime committed and the increase or decrease in that amount.
  2. Comment upon the movement of population in the U.S. as indicated in the census rates of mortality and immigration. Upon the movement of population in France and in other European countries during this century. Can you account for the decline in the rates of mortality which characterize these populations?
    Give an account of the growth of some of the large European cities and of the migratory movements of their populations. Can you account for the depopulation of rural districts which has taken place during this century?
  3. Give some account of the Descriptive School of Statisticians and of the School of Political Arithmetic.
    Of the organization and work of statistical bureaus in European countries during this century.
    Of the census bureau in the United States.

 

B.
[Take four.]

  1. What are some of the “positive” statistical evidences of vitality in a population? “negative”?
  2. Define “index of mortality.”
  3. Comment upon the density and distribution of population in the United States.
  4. What do you understand by “normal distribution of a population according to sex and age”? Define “movement of population.”
  5. Explain the various methods of estimating a population during intercensal years.

Source: Harvard University Archives. Harvard University Mid-year Examinations, 1852-1943. Box 4. Bound volume: Examination Papers, Mid-Years. 1897-98.

 

1897-98.
ECONOMICS 4.
End-year Examination.

Divide your time equally between A. and B.

A.

I.

“The wealth of a nation is a matter of estimate only. Certain of its elements are susceptible of being approximated more closely than others; but few of them can be given with greater certainty or accuracy than is expressed in the word ‘estimated.’” Why? State the several methods used for determining the wealth of a nation. Give some account of the increase and of the present distribution of wealth in the United States.

II.

What statistical data indicate the movement of real wages during this century? What facts have to be taken into account in determining statistically the condition of wage earners? State the several methods of calculating index numbers of wages and prices, and explain the merits of each method. Explain the use of weighted averages as indexes, and the considerations determining the weights. What has been the movement of wages and prices in the United States since 1860?

III.

Statistical data establishing a hierarchy of European races, the fundamental “laws of anthropo-sociology,” and the selective influences of migratory movements and the growth of cities.

 

B.
Take six.

  1. “I have striven with the help of biology, statistics and political economy to formulate what I consider to be the true law of population.” (Nitti.) What is this law? Is it the true law? Why?
  2. Upon what facts rests the assertion that “the fulcrum of the world’s balance of power has shifted from the West to the East, from the Mediterranean to the Pacific”?
  3. What factors determine the rate of suicide? Consider the effect upon the rate of suicide of the sex and age distribution of the population, of the social and physical environment, and of heredity.
  4. Statistical determination of labor efficiency, and the increase of such efficiency during this century.
  5. How far are statistics concerning the number of criminal offenders indicative of the amount of criminality? Statistics of prison populations? Of crimes? What variables enter in to determine the “rate of criminality”? What significance do you attach to such rates?
  6. The statistical method.
  7. Graphics as means of presenting statistical data.

Source: Harvard University Archives. Harvard University, Examination papers, 1873-1915. Box 5, Bound volume: Examination Papers 1898-99. Papers set for Final Examinations in History, Government, Economics, Philosophy, Fine Arts, Architecture, and Music in Harvard College. June 1898, pp.43-44.

_____________________

Course Enrollment 1898-99
(Year-course)

[Economics] 4. Dr. John Cummings. — Statistics. — Theory, method, and practice. — Studies in Demography. Lectures (3 hours) and conferences; 2 reports; theses.

Total 19: 10 Seniors, 4 Juniors, 4 Sophomores, 1 Other.

Source: Harvard University. Report of the President of Harvard College, 1898-1899, p. 73.

 

1898-99.
ECONOMICS 4.
Mid-Year Examination.

Devote one hour to A and the remainder of your time to B.

A.
Take two.

  1. The growth of modern cities and the laws governing the migrations of population as illustrated in the growth and constitution of the populations of London, Berlin, and other large cities.
  2. Define fully a “normal or life-table population,” considering its age and sex constitution and its movement.
  3. Discuss the development and predominance of the statistical method, and the gradual limitation of the field of statistical science.

B.
Take six.

  1. What do you understand by the “law of large numbers”?
    Discuss some of the principles which should govern the formation of statistical judgments.
  2. The “new law of population.”
  3. The value of criminal statistics and the nature of the statistical proofs that the value of punishments is over-estimated.
  4. “Several tests are employed to measure the duration of human life, and we are at present concerned to determine their precise value, and the relationship existing between them.” What are some of these tests, their precise value and inter-relationship?
  5. What is the nature of the statistical evidence that the “influx of the population from the country into London is in the main an economic movement”?
  6. The rate of mortality in urban an in rural populations.
  7. Decline in the rates of natality in the populations of Europe and the United States.

Source: Harvard University Archives. Harvard University, Mid-year Examination papers, 1852-1943. Box 5, Bound volume: Examination Papers. Mid-years, 1898-99.

 

 

1898-99.
ECONOMICS 4.
End-year Examination.

Devote at least one hour, but not more than one hour and a half, to A, and the remainder of your time to B.

A.

  1. Statistics of wages, manufactures, and capital in the eleventh census of the United States.
  2. Movement of population and the standard of living. Consider in connection with the growth of population and the movement of wages, prices, efficiency of labor and capital, the exploitation of new natural sources of power and wealth, and the relative movements of industrial groups.

B.
Take six.

  1. Average wages as an index of social condition.
  2. Statistical indexes of pauperism.
  3. What is the statistical basis for calculating the doubling period of a population and of what is that period an index?
  4. Define normal distribution of population (a) by sex, (b) by age.
  5. Show how the economic value of a population is affected by its age and sex distribution.
  6. To what extent may the prison population of the United States as given in the eleventh census be accepted as an index of criminality for the population of the United States?
  7. The growth of cities and the movement of population. Consider the effect of “urbanization” upon rates of criminality, natality, and mortality.

Source: Harvard University Archives. Harvard University, Examination papers, 1873-1915. Box 5, Bound volume: Examination Papers 1898-99. Papers set for Final Examinations in History, Government, Economics, Philosophy, Fine Arts, Architecture, and Music in Harvard College. June 1899, p.30.

_____________________

Course Enrollment 1899-1900
(Year-course)

[Economics] 4. Dr. John Cummings. — Statistics. — Theory, method, and practice. — Studies in Demography. Lectures (3 hours) and conferences; 2 reports; theses.

Total 10: 1 Graduate, 2 Seniors, 4 Juniors, 2 Sophomores, 1 Other.

Source: Harvard University. Report of the President of Harvard College, 1899-1900, p. 69.

 

1899-1900.
ECONOMICS 4.
Mid-Year Examination.

Devote one hour to A and the remainder of your time to B.

A.

  1. Urban growth and migration. Consider the sex and age distribution of migrants, the natural increase of urban and rural populations, and the causes of migration into urban centres. Illustrate by considering the actual conditions and movement in some one country or important urban centre.
  2. The data of criminal statistics as an index of amount of criminality. Consider the tables relating to crime in the United States census; the several statistical methods of dealing with crime and with the criminal classes; age, sex, and civil status as a factor in criminality; and the law of criminal saturation.

B.
Elect ten, and answer concisely.

  1. and 2. [counts as two questions]. Statistical measurements of agglomeration. Consider statistical methods of determining degree of concentration, also definition of the urban unit.

3. and 4. [counts as two questions]. Causes tending to make the rate of mortality lower for urban than for rural populations? causes tending to make it higher? the rate of natality?

  1. Methods of estimating population for intercensal years.
  2. Statistical laws and freedom of the will
  3. Define “life-table population.”
  4. Define carefully the following terms: “birth rate,” “rate of natality”; “rate of mortality”; death rate”; “rate of nuptialité”; “marriage rate”; index of mortality.”
  5. What do you understand by normal distribution of population by sex? by age? by civil status?
  6. Economic value of a population as effected by its age and sex distribution? by movement? by immigration?
  7. Of what statistical significance is the doubling period for any population?
  8. Can you account for the retardation in the rate of movement of population during this century?
  9. Tell when, if ever, the following terms are identical:—
    1. mean age at death.
    2. mean age of living.
    3. mean duration of life.
    4. expectation of life.

Source: Harvard University Archives. Harvard University, Mid-year Examination papers, 1852-1943. Box 5, Bound volume: Examination Papers. Mid-years, 1899-1900.

 

1899-1900.
ECONOMICS 4.
End-year Examination.

Divide your time equally between A and B.

A.

  1. Statistical methods of estimating wealth accumulated.
    Comment critically upon the census statistics of wealth accumulated in the United States.
  2. Statistical evidences of the progress of the working classes in the last half-century. Discuss the movement of wages and prices.
    What do you understand by “index figures,” “average wages,” “average prices,” “weighted averages”? Explain methods of weighting.
  3. The growth of cities and social election.

 

B.
Two questions may be omitted.

  1. How far are social conditions in a community revealed in the birth rate? the death rate? by the “index of mortality”? What do you understand by “movement of population”?
  2. In constructing a life table what correction must be made for abnormal age and sex distribution? Define “mortality,” “natality,” “expectation of life.” How should you calculate the “mean duration of life” from the census returns?
  3. The limit to the increase of population in the food supply? in other forms of wealth?
  4. Can you formulate any laws which will be true in general of the migrations of population?
  5. Methods of estimating population for intercensal years.
  6. Statistics of manufacturers in the United States census.
  7. How should you calculate the economic value of a population?
  8. Take one:—
    The rate of suicide as evidence of degeneration.
    The tables relating to crime in the Federal census of the United States.
  9. How far is it possible to give to moral and social facts a quantitative statement?

Source: Harvard University Archives. Harvard University, Examination papers, 1873-1915. Box 5, Bound volume: Examination Papers 1900-01. June 1900, p. 32.

Image Source: “A Tribute to Dr. Cummings” in School Life, Volume 22 (September 1936), p. 12.

Categories
Curriculum Economics Programs Fields Harvard Statistics

Harvard. Report on statistics and national income courses. Crum and Frickey, 1945

 

William Leonard Crum and Edward Frickey taught Harvard’s economic statistics courses in the 1930s and 1940s.  Paul Samuelson recounted his second semester (Spring 1936) as a graduate student following his previous semester’s worth of Crum: “…I was able to learn genuine modern statistics from E. B. Wilson, bypassing Edwin Frickey (who with Leonard Crum taught at Harvard courses against modern statistics!)” [On this, Roger E. Backhouse’s Vol I: Becoming Samuelson, 1915-1948, p.101].

Reading the following intradepartmental report on economic statistics courses and how to integrate national income and product accounting into the graduate curriculum that was written by a committee of two (Crum and Frickey), one discovers that even a decade after Samuelson’s experience, the proper preparation of “ink charts” was a subject that warranted faculty discussion.  Harvard Ph.D. Robert Solow later went to Columbia to play catch-up ball with respect to statistical analysis before starting his M.I.T. contract.  Harvard economics was a full generation behind the times with respect to statistical method at mid-20th century.

A 1947 Crum/Frickey  joint memo regarding preparation for taking the comprehensive field exam in statistics has been posted earlier.

______________________

6 March 1945

Report on the course offerings in Statistics, and in National Income

At the Department meeting of 13 February, 1945, the undersigned were named a committee to study course offerings and proposed offerings in Statistics and in National Income, discuss their findings with the Chairman, and report to the Department. Attached are the two reports: I, on Statistics; II, (page 10) on National Income.

W.L. Crum
Edwin Frickey

*  *  *  *  *  *  *  *  *  *  *  *  *

I. The Offering in Statistics

At the meeting of the Harvard Economics Department on 13 February, 1945. W. L. C. and E. F. sought opinions from colleagues as to additional instruction needed in statistics and as to changes needed in existing instruction. The following is in part a report of the informal discussion, in part an indication of what W. L. C. and E. F. think can advisedly be done. The present statement is preliminary; a more definitive report will be prepared, after consultation with H. H. B., for submission to the Department at a later meeting.

  1. Opinion was expressed that many of our graduate students show conspicuous lack of ability to present statistical material in the form of chart or table, for example, in theses. Instruction in statistics here has for several years relied upon capacity of students to learn by emulation—they have abundant opportunity to acquaint themselves with good statistical presentation, both tabular and graphic, in our courses in statistics and in the source materials of other courses. In course 21a, some instruction is incidentally given in orderly tabulation of limited sorts, but we make no attempt to teach students to prepare ink charts. Apparently, something more is needed; and three suggestions, perhaps all to be followed together, are made:
    1. By compressing some other parts of the work, we can include a small amount of instruction on presentation in course 21a. This should help put those graduate students who are required to take that course here on the right track.
    2. For students not required to take 21a, because they have had the “equivalent” elsewhere, one possibly helpful device is to require in course 121b a written report involving presentation in tabular and graphic form. Such report could be graded if sufficient funds are available to cover the grading, and the instructors could make a moderate effort to advise particular students about defects in their reports. The reports would presumably be required of all students in 121b, whether or not they had had 21a.
    3. The Department’s specialists in statistics could advise any graduate student, whose thesis involved matters of statistical presentation, concerning such matters. When the Department acquires a general research laboratory, with a regular supervisor, the supervisor could give such advice. In the meantime, the instructors in statistics could stand ready to give such advice in appropriate cases. The undersigned emphasize that this advice should be understood to concern presentation of statistical materials: they do not feel but they should be called upon ordinarily to advise such as student about sources of statistics for his thesis, or about the methods of analyzing the statistics, or about their interpretation. They have often given advice on such matters in certain cases, and will continue to do so, but take the stand that they should not be regarded as under the obligation to give such advice to all comers. The point is that; if the candidate proposes to write a statistical thesis in any field of economics, a vital part of his job is to obtain, analyze, and interpret his data. We see no reason why faculty specialists in statistics should make an extraordinary contribution to a thesis which happens to have quantitative aspects.
  2. Little emphasis appeared, in any opinions expressed, on the need for laboratory instruction in statistics in our graduate offering. Some suggestion was advanced that the “homework” type of problem task could helpfully be employed. W. L. C. and E. F. have a little faith that much could be accomplished in this way – the great advantage of the supervised laboratory is that the supervisor can get students actively started on the task and can catch and clear away difficulties as they arise. (We assume, of course, any problem work of this sort, in graduate courses, should be on an advanced – not elementary – level.) To meet this suggestion, we propose only that point A2 above be put into effect, and that the following change in present operations be considered. At present, course 121a includes two home-work problems, which stretch over several weeks, but are not graded and are not used as bases for specific advice to individual students. The proposed change is that these problems be handed in, and treated like the problem described in A2. (In these cases, as in that case, grading of the reports would be feasible if funds are available for the purpose.)

An emphatic suggestion was made that graduate students have the use of laboratory equipment, and be made welcome in the laboratory. We do not believe this can be managed with the laboratory facilities of course 21a. We note, however, that a moderate chance now exists that the University will presently provide the Department with a research laboratory in statistics, adequately equipped, and under competent supervision. If and when this is done, no difficulty will arise in making ample place for work by graduate students on any statistical tasks in which they may properly be interested. We remark that the arguments in favor of a general research laboratory in statistics are much more likely to bring conviction in responsible quarters that the argument, however strongly put, in favor of facilities merely for the occasional use of graduate students.

  1. Supposing we are to give an additional half graduate course in statistics, opinions pointed toward three alternatives:
    1. A course in theory, intermediate between course 121a and Prof. Wilson’s course 122b. This does not appear a good use of our manpower, for the election in such a course would inevitably be small, especially as the mathematics prerequisites would necessarily be much more severe than those – almost nil – on which we now limp through 121a.
    2. A further course was suggested – beyond 121b and perhaps alternating with it – in topics in the application of statistics to economic fields. Economics 121b now includes a selected list of such topics, which varies moderately from year to year; but it is by no means a comprehensive coverage of all even of the major possibilities. We could readily prepare an additional half course to be called 121c of further topics in the applied fields, and many students would probably like such a course. Such a course can be described as follows:

Economics 121c will be a half-course which might be entitled Topics in Applied Economic Statistics. Economics 21a or its equivalent will be a prerequisite. Properly qualified undergraduates may, with the consent of the instructor, be admitted to economics 121c.

Economics 121c will deal with statistical problems arising in connection with the use of basic statistical data in a selected list of economic topics. (As compared with 121b this course will lay more emphasis on the basic material and less emphasis on statistical theory.)

On each topic each student will be expected to familiarize himself with the immediate and the basic sources of the main materials, through actual examination of such materials, and to present a critical appraisal of these fundamental statistics. The instructor will give a succinct historical background – an outline of the principal work which is already been done on the topic. The instructor and the class will work out together conclusions as to what are the leading issues involved, and will consider what it is that statisticians are trying to measure and what they should be trying to measure.

Such topics as the following will be included:

Consumption
Commodity prices
Cost-of-living
Employment and unemployment
Wages
Money and Banking
Production and Trade (certain phases)
Balance of international payments
Public Finance

        1. The subordinate suggestion that, in this case, basic preparation for the oral exam and also the write-off field might consist of 121a and either 121b or 121c, was advanced. A strong objection to this appears in the fact that 121b, although made up largely of topics in applied statistics, now includes – and should continue to do so – certain topics which need to be covered by every general economist (we do not here have in mind the statistical specialist) who is to have “literacy” in the field of economic statistics today. Several of the “applied” topics now in 121b include in fact fundamental matters of statistical theory needed by all economists, and not elsewhere covered in our instruction. These include, for example: the theory of index numbers, statistical deflation, secular trends in business cycles, the basic theory of measuring production and income, and at least demand and cost curves not to mention more sophisticated matters of econometrics. These essentially theoretical topics in statistics should remain part of the basic graduate year course in statistics. (This goes also for our present topic of national income: even if the Department offer a course in that subject, the course will not be taken by all students, and all should have at least the brief survey now in 121b). For the foregoing reason, we emphatically urge that 121a and 121b stand as the basic year course in the field, and that the new course 121c be regarded as an additional – but not an alternative–half course.
        2. The subordinate suggestion at 121b and 121c be given in alternate years appears to fall for the same reason given in C2a.

 

    1. Instead of the course described under C2 suggestion was made that we introduce a course in administrative (we use this word provisionally, for want of a better) statistics – mainly, but not exclusively, governmental statistics. We have not outlined such a course in full, but can suggest its nature by indicating that it would emphasize the problems encountered in actually doing statistical work in government or private agencies. Such topics as the preparation and use of index numbers of prices and production; the compilation and use of data on employment and the labor force; statistics of farm production and operation; the gathering of and analysis of facts concerning trade, both foreign and domestic; financial data such as are developed by the treasury, the S. E. C, the F. R. B., and private agencies; statistics used in the analysis of particular enterprises; the rapidly developing field of quality control in industry, suggest themselves for inclusion. The nature of the course can also be indicated by somewhat loose contrast with the course described under C2 above: in that course, the point of view is of the user (economist, or other analyst) of statistics, and attention is given to the origin of the statistics only in so far as it is needed to guide and inform the user. In this course, the point of view is of the maker of statistics, and attention is given to the use of the statistics only in so far as it is needed to guide the maker in his work. This course would go far toward meeting the contention that our students, while well founded in statistical theory, are not ready to handle the kind of statistical tasks which they encounter in government or other research agencies.

At the moment we are not ready to choose between the courses described under C2 and C3, the former (and obviously the latter) being understood as in addition to, and not alternative to, 121b.

 

  1. No opinion was expressed concerning course 122b, and we think it should continue to be given in alternate years.

No opinion was offered concerning the content of course 121a. We have in mind some compression of one of the topics know given. This, plus the longer term under the peace-time schedule, will enable us to give more satisfactory attention to the topic of small samples.

We were commissioned to report also on national income. This is covered in a separate memorandum.

 

  1. We layout now, in tentative form and subject to revision by the Department, our recommendation as to the entire offering in statistics in the early post-war years.

21a. Substantially as at present, but with the change outlined in A1.

121a. Substantially as at present, but with the change outlined in B and the change noted in D.

121b. Substantially as at present, but with the change outlined in A2.

(Courses 121a and 121b to be regarded as the core of the preparation in the field of statistics, and to be recommended to the candidates for the general oral in statistics as the most helpful unit in their preparation.)

121c. A new half course, either that described under C2 or that under C3. To be open to graduate students who have had 21a or by consent of the instructor to those who have had the equivalent of 21a, and by consent of the instructor to properly qualified undergraduates who have had 21a.

122b. Substantially as at present, and to be given in alternate years as at present.

*  *  *  *  *  *  *  *  *  *  *  *  *

II. Offering in National Income

The suggestion is made that a half course, at the graduate level, in National Income be offered. The main purpose of such a course would be to give our students an extensive factual basis for their use of national income concepts and data in a wide range of our theoretical and applied fields. The course by itself could provide only a beginning for specialization the subject of national income for its own sake, and we do not understand that the Department contemplates recognizing the field in that subject.

While the course should be concerned primarily with the facts of national income, we understand that some attention could properly be given to the interpretation of those facts into their economic and social implications. Moreover, even to handle properly the factual side, the course would need give much attention to matters of definition and concept, matters which actually stand at the root of most of the “problems” of measuring national income and its chief constituents.

The core of the course would consist of the presentation, discussion, and criticism of the existing statistical facts on the national income and its constituents. These materials would presumably be limited to the United States; although some of the critical portions of the course, dealing with concepts and the like, would necessarily make large drafts on studies in certain other countries. Emphasis would be on the problems of measurement, the effectiveness and validity of the methods used, and the appropriateness of the results obtained as answers to questions posed by the economist.

In addition to the over-all aggregate of national income, viewed in real and money terms and in its variations over time, the course would examine the chief constituents of national income. These would include:

  1. Contributions to national income by various types of economic activity.
  2. Contributions from various geographical regions (much less is known on this.)
  3. Allocation, so far as it is known, to the several factors of production.
  4. Distribution according to size of income (money income) received by individuals.
  5. Distribution of income according to use: consumption expenditures of individuals (perishable, semi-durable), consumption through government, savings (by individuals, by enterprises, by government).
  6. Capital formation, and its relation to savings.
  7. Relation of taxes and public expenditures to the flow of income.

Your committee makes no recommendation as to the personnel to be assigned the task of conducting such a course. It does recommend: that the course be limited to graduate students, and to those advanced concentrators who receive permission from the instructor(s); that all students who take the course be required to have completed one half year course at the graduate level in economic theory and in statistics; that the course be given each year, rather than in alternate years; that the course be considered as a pro-seminar in statistics for the purpose of excuse – under our existing rules for reducing the oral examination to three fields – from the oral examination in statistics.

 

Source: Harvard University Archives. Department of Economics, Correspondence & Papers, 1902-1950. Box 23. Folder “Course Announcement 1945-46”.

Image Source: Crum and Frickey in the Harvard Class Album, 1942 and 1950.

Categories
Exam Questions Harvard Statistics Suggested Reading

Harvard. Final exam for course on national income accounting. Crum, 1938

 

William Leonard Crum (1894-1967) taught economic statistics at Harvard from 1923-1948 before finishing his career at the University of California, Berkeley. He taught an undergraduate one-semester course, “The National Income”, only twice. In the extensive but incomplete Harvard archival collection of course final examinations I have only been able to find the final for the second term of the 1937-38 academic year. Full course reading lists were not in the course syllabi and outlines collection, but the reading period assignments for both years could be found.

_________________

Course Enrollments,

1937-38

[Economics] 21bhf. Professor Crum — The National Income.

Total 7: 1 Graduate, 4 Seniors, 2 Juniors.

Source: Harvard University. Report of the President of Harvard College, 1937-38, p. 85.

 

1938-39

[Economics] 21bhf. Professor Crum — The National Income.

Total 3: 2 Graduates, 1 Senior.

Source: Harvard University. Report of the President of Harvard College, 1938-39, p. 98.

_________________

Reading Period

May 9- June 1, 1938

Economics 21b: Read either of the following:

Colin Clark, National Income and Outlay, Chs. I-V, and VII.
R. F. Martin, National Income and Its Elements (entire).

Source: Harvard University Archives. Syllabi, course outlines and reading lists in Economics, 1895-2003. Box 2, Folder “Economics, 1937-1938”.

 

May 8- May 31, 1939

Economics 21b: Choose one of the following:

National Industrial Conference Board, National Income in the United States, 1799-1938 (entire book).
Simon Kuznets, Commodity Flow and Capital Formation, National Bureau of Economic Research, 1938 (Part II and Part III).

Source: Harvard University Archives. Syllabi, course outlines and reading lists in Economics, 1895-2003. Box 2, Folder “Economics, 1938-1939”.

_________________

Final exam, 1938

HARVARD UNIVERSITY
Economics 21b2

(Omit two of the first six questions, and omit one of the last two)

  1. (a) Outline the main items, listing as plus and minus, which must be covered in estimating national income by the net value product method.
    (b) Discuss the chief theoretical and practical points relating to the estimated allowance for depreciation.
  2. (a) Comment upon the main problems encountered in determining the net value product of “government”, considered as an “industry”.
    (b) Name two chief “transfer” items, and indicate – for each – how it should be treated in national income estimates, and why.
  3. (a) Discuss with care the way in which the accounting practice relative to inventory valuation affects estimates of national income.
    (b) Comment upon the place of “additions to business surplus” in the simple concepts of national income. Indicate whether this surplus-additional item can be estimated directly, or only indirectly.
  4. (a) What is meant by “entrepreneurial withdrawals”, and on what basis are they in general estimated? Give your view of the validity of such estimates, with reasons.
    (b) To what extent do the methods customarily employed in estimating the distribution of national income according to particular categories give a satisfactory appraisal of any oneof the four main types discussed in economic theory – wages, interest, rent, profits?
  5. (a) Discuss the place of capital gains in national income estimates.
    (b) What is meant by capital formation? What are the leading obstacles to a satisfactory measurement thereof?
  6. (a) Given an online account of the relation between size of income (of individuals) and the main sources from which income is derived. How, in general, does the business cycle affect these relationships?
    (b) What is meant by real income? Name and discuss two chief obstacles to the measurement thereof.
  7. (Clark) Name, and common briefly upon, the chief differences in method of estimating national income, as between Great Britain and the United States.
  8. (Martin) Four main types of entrepreneurs are distinguished – farmers, retail-store proprietors, service establishment owners, professional practitioners. Comment upon the data available for estimating incomes of these groups, and give your views as to the validity of such estimates.

Source: Harvard University Archives. Harvard University. Final examinations, 1853-2001. Box 3, Folder “Final examinations, 1937-1938”.

Image source: Portrait of William Leonard Crum from the Harvard Class Album, 1946.